xamlibrary.files.wordpress.com · 2018-09-01 · 5 ©Vision IAS 17. The Indian Councils Act of 1909...

44

Transcript of xamlibrary.files.wordpress.com · 2018-09-01 · 5 ©Vision IAS 17. The Indian Councils Act of 1909...

Page 1: xamlibrary.files.wordpress.com · 2018-09-01 · 5 ©Vision IAS 17. The Indian Councils Act of 1909 provided for which of the following provisions? 1. Non-official majority in Central
Page 2: xamlibrary.files.wordpress.com · 2018-09-01 · 5 ©Vision IAS 17. The Indian Councils Act of 1909 provided for which of the following provisions? 1. Non-official majority in Central
Page 3: xamlibrary.files.wordpress.com · 2018-09-01 · 5 ©Vision IAS 17. The Indian Councils Act of 1909 provided for which of the following provisions? 1. Non-official majority in Central
Page 4: xamlibrary.files.wordpress.com · 2018-09-01 · 5 ©Vision IAS 17. The Indian Councils Act of 1909 provided for which of the following provisions? 1. Non-official majority in Central
Page 5: xamlibrary.files.wordpress.com · 2018-09-01 · 5 ©Vision IAS 17. The Indian Councils Act of 1909 provided for which of the following provisions? 1. Non-official majority in Central

5 www.visionias.in ©Vision IAS

17. The Indian Councils Act of 1909 provided

for which of the following provisions?

1. Non-official majority in Central

Legislative Council.

2. Association of Indians with the

executive council of the Viceroy.

3. Separate electorate for Anglo-Indians

and Europeans.

Select the correct answer using the code

given below.

(a) 1 and 2 only

(b) 1 and 3 only

(c) 2 only

(d) 2 and 3 only

18. Which of the following Supreme Court cases

is/are related to disputes/conflicts between

Fundamental Rights and Directive Principles

of State Policy?

1. IR Coehlo Case, 2007

2. Golaknath case, 1967

3. Maneka Gandhi case, 1980

4. Minerva Mill Case, 1980

Select the correct answer using the code

given below.

(a) 1 and 2 only

(b) 2 and 4 only

(c) 3 and 4 only

(d) 1 only

19. Under Article 360 of the Constitution, the

power to proclaim financial emergency is

vested under

(a) Ministry of Finance

(b) Parliament

(c) President

(d) Comptroller and Auditor General of

India

20. Which among the following is a fundamental

duty under the constitution of India?

(a) To support bona fide civil society

movements.

(b) To keep the streets, roads and other

infrastructure of India's cities clean.

(c) To defend the country and render

national service when called upon to do

so

(d) To pay taxes.

21. According to the Indian Constitution, which

among the following categories of persons

would not be eligible to become the citizen

of India?

(a) who is resident in India for five years

immediately before independence.

(b) who migrated to Pakistan from India

after March 1, 1947, but later returned to

India for resettlement.

(c) whose parents were born in undivided

India and who migrated to India from

Pakistan before July 19, 1948.

(d) whose grandparents were born in

undivided India but person is residing

outside India.

22. Which of the following writs is issued by a

higher court to a lower court to transfer a

case pending with the latter to itself?

(a) Mandamus

(b) Certiorari

(c) Quo-Warranto

(d) Prohibition

Page 6: xamlibrary.files.wordpress.com · 2018-09-01 · 5 ©Vision IAS 17. The Indian Councils Act of 1909 provided for which of the following provisions? 1. Non-official majority in Central

6 www.visionias.in ©Vision IAS

23. With reference to the Central Vigilance

Commission, consider the following

statements:

1. It is a constitutional body.

2. Its Chairman and members are appointed

by the President.

Which of the statements given above is/are

correct?

(a) 1 only

(b) 2 only

(c) Both 1 and 2

(d) Neither 1 nor 2

24. Consider the following statements regarding

reorganisation of states :

1. States Reorganisation Commission of

1953 accepted the theory of 'one

language-one state'.

2. Gujarat was the first state to be created

on linguistic basis.

Which of the statements given above is/are

correct?

(a) 1 only

(b) 2 only

(c) Both 1 and 2

(d) Neither 1 nor 2

25. With reference to the Regulating Act of

1773, consider the following statements:

1. It recognised the political and

administrative functions of the East

India Company.

2. It laid the foundations of central

administration in India.

Which of the statements given above is/are

correct?

(a) 1 only

(b) 2 only

(c) Both 1 and 2

(d) Neither 1 nor 2

26. With reference to the Constitution of India,

the National Emergency constitutes

limitation upon

1. Executive power of Centre

2. Fundamental Rights under Article 19

Select the correct answer using the code

given below.

(a) 1 only

(b) 2 only

(c) Both 1 and 2

(d) Neither 1 nor 2

27. Which of the following is/are correct with

reference to the right to property?

1. It is a legal right.

2. It can be curtailed by an ordinary law of

the Parliament.

3. In case of violation, the aggrieved

person can directly move the Supreme

Court for its enforcement.

Select the correct answer using the code

given below.

(a) 1 and 2 only

(b) 3 only

(c) 1 only

(d) 1, 2 and 3

28. With reference to the National Commission

for Scheduled Castes, consider the following

statements:

1. It investigates all matters relating to the

constitutional and other legal safeguards

for the Anglo-Indian community.

2. For matters relating to state government,

the Commission submits the report to

the concerned state Governor.

Which of the statements given above is/are

correct?

(a) 1 only

(b) 2 only

(c) Both 1 and 2

(d) Neither 1 nor 2

Page 7: xamlibrary.files.wordpress.com · 2018-09-01 · 5 ©Vision IAS 17. The Indian Councils Act of 1909 provided for which of the following provisions? 1. Non-official majority in Central

7 www.visionias.in ©Vision IAS

29. With reference to citizen’s right to equality

of opportunity in public employment, which

of the following is/are prohibited under the

constitution?

1. Parliament prescribing residence as a

condition for certain employment.

2. Reservation of posts in favour of any

backward class that is not adequately

represented in the state services.

Select the correct answer using the code

given below.

(a) 1 only

(b) 2 only

(c) Both 1 and 2

(d) Neither 1 nor 2

30. Which of the following principles were

included in the 'Objectives Resolution'

passed by the Constituent Assembly?

1. Powers and authority of independent

India and its constitution shall flow from

the people.

2. Minorities and Other backward classes

will be provided adequate safeguards.

3. India will make full and willing

contribution to the promotion of world

peace.

Select the correct answer using the code

given below

(a) 1, 2 and 3 only

(b) 2 and 3 only

(c) 1 and 2 only

(d) 1 only

31. With reference to Advocate General of State,

consider the following statements:

1. He is appointed by Governor.

2. He must be at least 35 years of age to be

appointed as Advocate General of State.

3. His remuneration is decided by

Governor.

Which of the statements given above are

correct?

(a) 1 and 2 only

(b) 2 and 3 only

(c) 1 and 3 only

(d) 1, 2 and 3

32. Which of the following are the federal

features of Constitution?

1. Division of powers

2. Written Constitution

3. Supremacy of Constitution

4. Rigidity of Constitution

5. Independent judiciary

6. Bicameralism

Select the correct answer using the code

given below.

(a) 1, 2 and 3 only

(b) 3, 4, 5 and 6 only

(c) 1, 2, 3, 4 and 6 only

(d) 1, 2, 3, 4, 5 and 6

Page 8: xamlibrary.files.wordpress.com · 2018-09-01 · 5 ©Vision IAS 17. The Indian Councils Act of 1909 provided for which of the following provisions? 1. Non-official majority in Central

8 www.visionias.in ©Vision IAS

33. Which of the following protections is/are

available as a fundamental right under the

constitution to a person detained under a

preventive detention law?

1. The grounds of detention should be

communicated to the detenu even if it is

against the public interest.

2. The detenu must be released after 24

hours unless the magistrate authorises

further detention.

3. The detenu must be offered an

opportunity to make a representation

against the detention order.

Select the correct answer using the code

given below.

(a) 3 only

(b) 2 and 3 only

(c) 2 only

(d) 1 and 3 only

34. Which one of the following best describes

the term/expression 'Bill of Rights'?

(a) Legislative proposal to add to the

fundamental rights in the Constitution.

(b) A list of rights mentioned and protected

by the Constitution.

(c) An assessment of the cost incurred by

the judiciary in hearing matters related

to fundamental rights.

(d) A list of duties citizens must perform in

order to qualify for enjoyment of

fundamental rights.

35. With reference to the Lokpal and

Lokayuktas Act (2013), consider the

following statements:

1. The Lokpal consists of a Chairperson

with a maximum of 8 members of which

50% shall be judicial members.

2. The Lokpal has the power of

superintendence and direction over the

CBI for cases referred to them by the

Lokpal.

Which of the statements given above is/are

correct?

(a) 1 only

(b) 2 only

(c) Both 1 and 2

(d) Neither 1 nor 2

36. Which of the following provisions were

added to the Constitution through

Constitutional amendments?

1. The President has to act in accordance

with the advice of Council of Ministers.

2. All citizens who are not less than 18

years old are entitled to vote in elections

of Lok Sabha.

3. The State can make special provisions

for the advancement of socially and

educationally backward classes.

Select the correct answer using the code

given below.

(a) 1 and 2 only

(b) 1 and 3 only

(c) 2 and 3 only

(d) 1, 2 and 3

Page 9: xamlibrary.files.wordpress.com · 2018-09-01 · 5 ©Vision IAS 17. The Indian Councils Act of 1909 provided for which of the following provisions? 1. Non-official majority in Central

9 www.visionias.in ©Vision IAS

37. With reference to the Comptroller and

Auditor General of India, which of the

following statements is/are correct?

1. He is the head of the Indian Audit and

Accounts Department.

2. He holds office for a period of five years

or upto the age of 70 years, whichever is

earlier.

3. The Union Finance minister can

represent the CAG in Parliament during

his absence.

Select the correct answer using the code

given below.

(a) 1 only

(b) 2 and 3 only

(c) 1 and 2 only

(d) 1, 2 and 3

38. The Indian Independence Act of 1947

abolished the office of the secretary of state

of India and transferred its functions to:

(a) Prime Minister of Britain

(b) Secretary of state for Commonwealth

Affairs

(c) Governor-General of India

(d) First Cabinet of Free India

39. Which of the following Directive Principles

is based on the Gandhian ideology?

(a) To promote cottage industries on an

individual or co-operation basis in rural

areas.

(b) To organise agriculture and animal

husbandry on modern and scientific

lines.

(c) To raise the level of nutrition and the

standard of living of people and to

improve public health.

(d) All of the given Directive Principles are

based on Gandhian ideology.

40. Right to form cooperative societies in India

is a

(a) Fundamental Right

(b) Legal Right

(c) Constitutional Right

(d) Natural Right

41. With reference to the Special Officer for

Linguistic Minorities, consider the following

statements:

1. It was constituted under Article 350-B of

the Indian Constitution.

2. It can investigate all matters relating to

the safeguards provided for both

linguistic and religious minorities under

the Constitution.

Which of the statements given above is/are

correct?

(a) 1 only

(b) 2 only

(c) Both 1 and 2

(d) Neither 1 nor 2

42. According to the Preamble of the

Constitution, as adopted by the Constituent

Assembly, which of the following adjectives

was/were used to define the Indian

Republic?

1. Socialist

2. Sovereign

3. Secular

4. Democratic

Select the correct answer using the code

given below.

(a) 1, 2 and 3 only

(b) 2 and 4 only

(c) 1 and 4 only

(d) 1, 2, 3 and 4

Page 10: xamlibrary.files.wordpress.com · 2018-09-01 · 5 ©Vision IAS 17. The Indian Councils Act of 1909 provided for which of the following provisions? 1. Non-official majority in Central

10 www.visionias.in ©Vision IAS

43. With reference to the National Human

Rights Commission, consider the following

statements:

1. It consists only of members who have

held judicial office.

2. It does not have the power of

prosecution.

3. It can inquire suo-moto into any

violation of human rights.

Which of the statements given above is/are

correct?

(a) 1 and 2 only

(b) 2 and 3 only

(c) 3 only

(d) 1, 2 and 3

44. With reference to the provisions about

fundamental duties in the Indian

Constitution, consider the following

statements:

1. They include both civic duties and moral

duties.

2. Parliament can provide for the

imposition of appropriate penalty for

failure to fulfil any of them.

3. They do not apply to foreigners.

Which of the statements given above is/are

correct?

(a) 2 only

(b) 1 and 2 only

(c) 3 only

(d) 1, 2 and 3

45. Which of the following is/are correct with

reference to a Constitution?

1. It determines the relationship among

citizens and that between citizens and

government.

2. It lays down limits on the powers of the

government.

Select the correct answer using the code

given below.

(a) 1 only

(b) 2 only

(c) Both 1 and 2

(d) Neither 1 nor 2

46. With reference to Election Commission,

consider the following statements:

1. Conditions of office and tenure of office

are provided under Article 324.

2. Chief Election Commissioner (CEC)

exercises veto power over the matters

discussed in Election Commission.

3. CEC does not hold office during

pleasure of President.

Which of the statements given above is/are

correct?

(a) 1 and 2 only

(b) 3 only

(c) 1 and 3 only

(d) 1, 2 and 3

47. Who among the following is/are not eligible

for further office either in center or state

after they ceases to hold their respective

offices?

1. Comptroller and Auditor General of

India

2. Attorney General of India

3. Members of Union Public Service

Commission

Select the correct answer using the code

given below.

(a) 1 and 2 only

(b) 1 only

(c) 2 and 3 only

(d) 3 only

48. With reference to Article 12 of the

Constitution, the term ‘State’ includes which

of the following?

1. Parliament and legislature of states.

2. Union executive and executive of the

state governments.

3. Municipalities and Panchayats

4. Authorities like LIC and ONGC.

Select the correct answer using the code

given below.

(a) 1 and 3 only

(b) 2 and 4 only

(c) 1, 2 and 3 only

(d) 1, 2, 3 and 4

Page 11: xamlibrary.files.wordpress.com · 2018-09-01 · 5 ©Vision IAS 17. The Indian Councils Act of 1909 provided for which of the following provisions? 1. Non-official majority in Central

11 www.visionias.in ©Vision IAS

49. Consider the following statements:

1. The parliamentary system is based on

the doctrine of separation of powers

between the legislature and executive.

2. The presidential system is based on the

principle of cooperation and co-

ordination between the legislative and

executive organs.

Which of the statements given above is/are

not correct?

(a) 1 only

(b) 2 only

(c) Both 1 and 2

(d) Neither 1 nor 2

50. A State Human Rights Commission can

inquire into violations of human rights in

respect of subjects mentioned in which of

the following lists of the seventh schedule of

the Constitution?

1. State List

2. Concurrent List

3. Union List

Select the correct answer using the code

given below.

(a) 1 only

(b) 1 and 2 only

(c) 1, 2 and 3

(d) None

51. Consider the following statements:

1. The term ‘Federation’ has nowhere been

mentioned in the Constitution.

2. No state has the right to secede from the

federation.

Which of the statement given above is/are

correct?

(a) 1 only

(b) 2 only

(c) Both 1 and 2

(d) Neither 1 nor 2

52. With reference to the Finance Commission,

consider the following statements:

1. It is a quasi-judicial body.

2. The members hold office for five years

or till 65 years of age.

3. The members are eligible for

reappointment.

Which of the statements given above is/are

correct?

(a) 1 and 2 only

(b) 2 only

(c) 1 and 3 only

(d) 1 only

53. With reference to the theory of 'basic

structure', consider the following statements:

1. It first emerged in the Kesavananda

Bharati case.

2. It weakens the balance between rigidity

and flexibility of the Indian Constitution.

Which of the statements given above is/are

correct?

(a) 1 only

(b) 2 only

(c) Both 1 and 2

(d) Neither 1 nor 2

54. Which of the following is 'Cultural and

Educational' Rights regarded by the Indian

Constitution?

(a) The State shall provide free and

compulsory education to all children of

the age of six to fourteen years in such a

manner as the State may determine.

(b) Freedom as to attendance at religious

instruction or religious worship in

certain educational institutions.

(c) Any section of the citizens residing in

the territory of India or any part having a

distinct language, script or culture of its

own shall have the right to conserve the

same.

(d) Duty of the Union to promote the spread

of the Hindi language and to develop it

so that it may serve as a medium of

expression for all the elements of the

composite culture of India.

Page 12: xamlibrary.files.wordpress.com · 2018-09-01 · 5 ©Vision IAS 17. The Indian Councils Act of 1909 provided for which of the following provisions? 1. Non-official majority in Central

12 www.visionias.in ©Vision IAS

55. Which among the following constitutional

amendments fall outside the scope of Article

368?

1. Termination of Citizenship

2. Creation of legislative councils in states

3. Inclusion of new Directive Principles of

State Policy.

Select the correct answer using the code

given below.

(a) 3 only

(b) 2 and 3 only

(c) 1 only

(d) 1 and 2 only

56. Which of the following rights are conferred

only to the Citizens of India and denied to

aliens?

1. Right to contest for the membership of

the Parliament.

2. Freedom of speech and expression.

3. Right to elementary education.

Select the correct answer using the code

given below.

(a) 1 only

(b) 2 and 3 only

(c) 1 and 2 only

(d) 1, 2 and 3

57. Which of the following Directive Principles

was added by 42nd Constitution

amendment?

1. To secure opportunities for healthy

development of children.

2. To promote equal justice and to provide

free legal aid to the poor.

3. To minimise inequalities in income,

status, facilities and opportunities.

4. To promote development of Hindi

language.

Select the correct answer using the code

given below.

(a) 1 and 2 only

(b) 2 and 3 only

(c) 1, 2 and 4 only

(d) 1, 2, 3 and 4

58. With reference to the Article 14 of the Indian

Constitution, consider the following

statements:

1. It lays down the foundation of ‘Rule of

Law’ in India.

2. It is the only fundamental right which is

absolute in nature.

Which of the statements given above is/are

correct?

(a) 1 only

(b) 2 only

(c) Both 1 and 2

(d) Neither 1 nor 2

59. Consider the following pairs:

Schedule Subject

1. Third Schedule : Emoluments,

Allowances and

Privileges

2. Fourth Schedule : Administration

and control of

scheduled areas

and scheduled

tribes

3. Ninth Schedule : Disqualification

of member of

Parliament and

State legislatures

Which of the pairs given above is/are

correctly matched?

(a) 1 only

(b) 2 only

(c) 2 and 3 only

(d) None

Page 13: xamlibrary.files.wordpress.com · 2018-09-01 · 5 ©Vision IAS 17. The Indian Councils Act of 1909 provided for which of the following provisions? 1. Non-official majority in Central

13 www.visionias.in ©Vision IAS

60. With reference to the State Information

Commission, consider the following

statements:

1. The State Chief Information

Commissioner and State Information

Commissioners are appointed by the

Governor.

2. To remove a State Information

Commissioner on the ground of proved

misbehaviour and incapacity, the

Governor has to refer the matter to the

High Court for an enquiry.

Which of the statements given above is/are

correct?

(a) 1 only

(b) 2 only

(c) Both 1 and 2

(d) Neither 1 nor 2

61. The chairpersons of which of the following

bodies are ex-officio members of the

National Human Rights Commission?

1. National Commission for STs

2. National Development Council (NDC)

3. Office of the Special Officer for

Linguistic Minorities

4. National Commission for Women

Select the correct answer using the code

given below.

(a) 2, 3 and 4 only

(b) 1, 3 and 4 only

(c) 1 and 4 only

(d) 1, 2 and 3 only

62. NITI Aayog was formed in 2015 by:

(a) an ordinance promulgated by the

President.

(b) a resolution of the Central Government.

(c) a law of the Parliament.

(d) an amendment to the Constitution under

Article 368.

63. Consider the following statements:

1. An amendment of the Constitution can

be initiated only by the prior permission

of the President.

2. All Constitutional Amendment bills

must be passed in each House by a

simple majority.

Which of the statements given above is/are

correct?

(a) 1 only

(b) 2 only

(c) Both 1 and 2

(d) Neither 1 nor 2

64. The Fundamental Duties in the Indian

Constitution were inspired by the

Constitution of

(a) USSR

(b) USA

(c) Germany

(d) France

65. Which of the following articles of the

Constitution explicitly provides for the

doctrine of judicial review?

(a) Article 13

(b) Article 32

(c) Article 50

(d) Article 226

66. On which of the following matters do the

state legislatures have the power to make

laws?

1. Prescribing punishment for forced

labour.

2. Restricting fundamental rights of

members of police forces.

3. Prescribing residence as a condition for

certain employments in a state

Select the correct answer using the code

given below.

(a) 1 only

(b) 2 and 3 only

(c) 2 only

(d) None

Page 14: xamlibrary.files.wordpress.com · 2018-09-01 · 5 ©Vision IAS 17. The Indian Councils Act of 1909 provided for which of the following provisions? 1. Non-official majority in Central

14 www.visionias.in ©Vision IAS

67. With reference to State Election

Commission, consider the following

statements:

1. It has been empowered to conduct

elections to panchayats through 73rd

Constitutional amendment.

2. The functions of delimitation,

reservation and rotation of Panchayats

seats is vested in State Election

Commission.

3. State Election Commission submits its

annual report to the Election

Commission of India and to the

Governor.

Which of the statements given above is/are

correct?

(a) 1 and 2 only

(b) 2 only

(c) 1 and 3 only

(d) 1, 2 and 3

68. Which of the following languages are not

included in the Eight Schedule of

Constitution?

1. Nepali

2. English

3. Rajasthani

4. Konkani

5. Urdu

6. Bhojpuri

Select the correct answer using the code

given below.

(a) 2, 3 and 6 only

(b) 3 and 5 only

(c) 1, 5 and 6 only

(d) 1, 2, 5 and 6

69. Consider the following statements regarding

the differences between Fundamental Rights

and Legal /ordinary rights:

1. Unlike the legal rights, it is easier to

amend the Fundamental rights to meet

the changing requirements of citizens.

2. Unlike the legal rights, reasonable

restrictions can not be imposed on

Fundamental rights as they are

guaranteed and protected by the

Constitution.

Which of the statements given above is/are

correct?

(a) 1 only

(b) 2 only

(c) Both 1 and 2

(d) Neither 1 nor 2

70. The term 'law' defined under the Article 13

of the Constitution inlcudes which of the

following?

(a) Ordinary laws

(b) Constitutional Amendment Acts

(c) Customs of people having force of law.

(d) Both (a) and (c)

71. Which of the following is/are features of

Indian secularism as adopted in the

Constitution?

1. Religion is a personal matter and there is

no interference by the State.

2. No citizen can be denied entry into any

educational institute of the state on the

grounds of religion or caste.

3. Every religious denomination has the

right to establish and maintain charitable

institutions without any limitation.

Select the correct answer using the code

given below.

(a) 1 only

(b) 1 and 2 only

(c) 2 only

(d) 2 and 3 only

Page 15: xamlibrary.files.wordpress.com · 2018-09-01 · 5 ©Vision IAS 17. The Indian Councils Act of 1909 provided for which of the following provisions? 1. Non-official majority in Central

15 www.visionias.in ©Vision IAS

72. Writ of Mandamus cannot be issued against

which of the following?

1. Private individual

2. President

3. Tribunals

Select the correct answer using the code

given below.

(a) 2 and 3 only

(b) 2 only

(c) 1 and 2 only

(d) 1 only

73. Consider the following statements.

1. Revocation of President’s rule in a state

requires parliamentary approval.

2. 44th Amendment Act of 1978 restrained

the power of Parliament to extend a

proclamation of President’s Rule.

Which of the statements given above is/are

correct?

(a) 1 only

(b) 2 only

(c) Both 1 and 2

(d) Neither 1 nor 2

74. With reference to the power of courts to

issue writs, consider the following

statements:

1. Constitution confers the power to issue

writs to the Supreme Court, the High

Courts and the District Courts.

2. The Supreme Court can issue writs not

only for the enforcement of Fundamental

Rights but also for enforcement of an

ordinary legal right.

Which of the statements given above is/are

correct?

(a) 1 only

(b) 2 only

(c) Both 1 and 2

(d) Neither 1 nor 2

75. With regard to Freedom of Speech and

Expression guaranteed under Indian

constitution, Reasonable restrictions

1. are not justiciable in nature.

2. covers both public and private interest.

Which of the statements given above is/are

correct?

(a) 1 only

(b) 2 only

(c) Both 1 and 2

(d) Neither 1 nor 2

76. Which of the following are non-

constitutional bodies?

1. National Commission for STs

2. NITI Aayog

3. Central Bureau of Investigation

Select the correct answer using the code

given below.

(a) 1 and 2 only

(b) 1 and 3 only

(c) 1, 2 and 3

(d) 2 and 3 only

77. Which of the following fall within the

purview of Freedom of speech and

expression as guaranteed under article 19 of

Indian constitution?

1. Freedom of Silence

2. Right to Strike

3. Right to telecast

4. Right to know about government

activities

Select the correct answer using the code

given below.

(a) 1, 3 and 4 only

(b) 4 only

(c) 2 and 3 only

(d) 1, 2, 3 and 4

Page 16: xamlibrary.files.wordpress.com · 2018-09-01 · 5 ©Vision IAS 17. The Indian Councils Act of 1909 provided for which of the following provisions? 1. Non-official majority in Central

16 www.visionias.in ©Vision IAS

78. Consider the following statements:

1. The citizens of India owe allegiance to

both the Union and State.

2. In India, only a citizen by birth is

eligible for the office of President.

Which of the statements given above is/are

correct?

(a) 1 only

(b) 2 only

(c) Both 1 and 2

(d) Neither 1 nor 2

79. Which of the following statements is/are

correct regarding the Constituent Assembly?

1. It was not a fully sovereign body.

2. It was directly elected by the people

3. It became the first parliament of India.

Select the correct answer using the code

given below.

(a) 1 and 3 only

(b) 1 and 2 only

(c) 3 only

(d) 2 only

80. Consider the following statements:

1. There is no limit to the power of the

Parliament to amend the Constitution.

2. Only the President can decide whether

any matter forms a part of the basic

structure of the Constitution

Which of the statements given above is/are

correct?

(a) 1 only

(b) 2 only

(c) Both 1 and 2

(d) Neither 1 nor 2

81. With reference to Joint Public Service

Commission (JPSC), consider the following

statements:

1. It can be created by President on the

request of state legislatures concerned.

2. Its members holds office for a term of

six years or until they attain the age of

62 years, whichever is earlier.

3. It presents its annual report to the

President.

Which of the statement given above are

not correct?

(a) 1 and 2 only

(b) 2 and 3 only

(c) 1 and 3 only

(d) 1, 2 and 3

82. With reference to Attorney General of India,

consider the following statements:

1. He must be a person who is qualified to

be appointed as a judge of the Supreme

Court.

2. He receives such remuneration as the

Parliament may determine.

3. He can take part in the proceedings of

both the Houses of Parliament or their

joint sitting.

Which of the statement given above is/are

correct?

(a) 1 and 2 only

(b) 2 only

(c) 3 only

(d) 1 and 3 only

Page 17: xamlibrary.files.wordpress.com · 2018-09-01 · 5 ©Vision IAS 17. The Indian Councils Act of 1909 provided for which of the following provisions? 1. Non-official majority in Central

17 www.visionias.in ©Vision IAS

83. Which among the following functions

was/were performed by the the Constituent

Assembly?

1. Ratification of India's membership of the

Commonwealth.

2. Adoption of National flag and National

Anthem.

3. Election of Dr. Rajendra Prasad as the

first President of India.

Select the correct answer using the code

given below.

(a) 1 only

(b) 1 and 2 only

(c) 2 and 3 only

(d) 1, 2 and 3

84. Which of the following would reflect an

appropriate relationship between equality

and liberty?

1. Liberty without equality would produce

the supremacy of few over many.

2. Equality without liberty would kill

individual initiative.

Select the correct answer using the code

given below.

(a) 1 only

(b) 2 only

(c) Both 1 and 2

(d) Neither 1 nor 2

85. Consider the following pairs:

Committee of

the Constituent

Assembly

Headed by

1. Steering

Committee

: Dr. Rajendra

Prasad

2. Committee for

negotiating with

States

: Sardar Patel

3. Union

Constitution

Committee

: Jawaharlal

Nehru

Which of the pairs given above are correctly

matched?

(a) 1 and 2 only

(b) 1 and 3 only

(c) 2 and 3 only

(d) 1, 2 and 3

86. Consider the following pairs:

Commission/Body Falls under

1. National

Commission for

Minorities

: Ministry of

Social Justice

&

Empowerment

2. National

Commission for

Women

: Ministry of

Women and

Child

Development

3. National

Commission for

STs

: Ministry of

Tribal Affairs

Which of the pairs given above is/are

correctly matched?

(a) 3 only

(b) 1 and 2 only

(c) 2 and 3 only

(d) 1, 2 and 3

87. The Charter Act of 1833 led to:

1. deprivation of legislative powers of

Governors of Bombay and Madras.

2. separate body for commercial and

administrative functions of the East

India Company.

3. introduction of an open competitive

system of selection and recruitment of

civil servants.

Select the correct answer using the code

given below.

(a) 1 only

(b) 1 and 3 only

(c) 2 only

(d) 1, 2 and 3

88. Article 15 of the Constitution provides that

the State cannot discriminate against any

citizen on grounds only of

1. Caste

2. Place of birth

3. Language

4. Religion

Select the correct answer using the code

given below.

(a) 1 and 2 only

(b) 3 and 4 only

(c) 1, 2 and 4 only

(d) 1, 2, 3 and 4

Page 18: xamlibrary.files.wordpress.com · 2018-09-01 · 5 ©Vision IAS 17. The Indian Councils Act of 1909 provided for which of the following provisions? 1. Non-official majority in Central

18 www.visionias.in ©Vision IAS

89. Consider the following statements:

1. The conversion of the federal structure

of India into a unitary one during

emergency has been borrowed from the

Weimar Constitution of Germany.

2. The suspension of Fundamental Rights

during emergency is a unique feature of

the Indian Constitution.

Which of the statements given above is/are

correct?

(a) 1 only

(b) 2 only

(c) Both 1 and 2

(d) Neither 1 nor 2

90. If any foreign territory becomes a part of

India, then the power to specify that the

people of a foreign territory shall be the

citizens of India is vested in the

(a) National Human Rights Commission

(b) Supreme Court

(c) Union Government

(d) Parliament

91. Consider the following statements:

1. The expression 'Union of India' under

Article 1 of the Indian constitution

includes those territories that India may

accquire in future.

2. India can acquire foreign territories

through cession only.

Which of the statements given above is/are

correct?

(a) 1 only

(b) 2 only

(c) Both 1 and 2

(d) Neither 1 nor 2

92. A citizen may lose his/her citizenship when

he/she

1. voluntarily acquires the citizenship of

any other foreign state.

2. unlawfully traded or communicated with

the enemy during a war.

3. obtained the citizenship by fraud.

Select the correct answer using the code

given below.

(a) 1 and 2 only

(b) 1 and 3 only

(c) 2 and 3 only

(d) 1, 2 and 3

93. Which of the following provisions have been

regarded as 'heart and soul of the

constitution' by Dr. B.R. Ambedkar?

1. Right to Life

2. Directive Principles of State Policy

3. Right to Constitutional Remedies

Select the correct answer using the code

given below.

(a) 2 and 3 only

(b) 1 and 2 only

(c) 3 only

(d) 2 only

94. With reference to the Chief Information

Commissioner, consider the following

statements:

1. His term of office is five years.

2. He should not hold any other office of

profit or pursue any profession.

3. He is not eligible for reappointment.

Which of the statements given above is/are

correct?

(a) 1 only

(b) 2 and 3 only

(c) 1 and 3 only

(d) 1, 2 and 3

Page 19: xamlibrary.files.wordpress.com · 2018-09-01 · 5 ©Vision IAS 17. The Indian Councils Act of 1909 provided for which of the following provisions? 1. Non-official majority in Central

19 www.visionias.in ©Vision IAS

95. India has been using the Constitution as the

basic framework within which the

government operates. Which of the

following are not the reasons for it?

(a) The judiciary and political practice have

shown flexibility in the implementation

of the Constitution.

(b) The basic framework of the Constitution

suits our country.

(c) The Constitution makers were very far

sighted and provided for solutions for

many situations.

(d) The Constitution of India is a sacred

document and cannot be changed.

96. Which of the following is/are common to

both ‘interest groups’ and ‘people’s

movements’?

1. Both attempt to influence politics

without directly taking part in electoral

competition.

2. Both have a loose organisation, where

decision making is informal and flexible.

3. Both are largely dependent on

spontaneous mass participation.

Select the correct answer using the code

given below.

(a) 1 only

(b) 2 and 3 only

(c) 1 and 3 only

(d) 1, 2 and 3

97. The Indian Constitution guarantees which of

the following to the citizens?

1. Equality of status

2. Equality of opportunities

3. Equality of outcomes

4. Equality before law

Select the correct answer using the code

given below.

(a) 1, 2 and 3 only

(b) 2, 3 and 4 only

(c) 1, 3 and 4 only

(d) 1, 2 and 4 only

98. Which of the following legislations have

provisions that implement some of the

Fundamental Duties?

1. The Indian Penal Code (IPC)

2. The Representation of People Act

(1951)

3. Forest (Conservation) Act of 1980

Select the correct answer using the code

given below.

(a) 1 and 2 only

(b) 2 and 3 only

(c) 1 and 3 only

(d) 1, 2 and 3

99. With reference to UPSC, which of the

following statements is/are correct?

1. It is mandated with the classification of

services and cadre management of

selected candidates.

2. It presents its annual performance report

to the Parliament.

3. Its jurisdiction can be extended by an act

made by the Parliament.

Select the correct answer using the code

given below.

(a) 1 and 2 only

(b) 2 and 3 only

(c) 3 only

(d) none

100. Even though the Indian Parliamentary

System is largely based on the British

pattern, which of the following is/are the

fundamental differences between the two?

1. The Indian Parliament is not a sovereign

body like the British Parliament.

2. While the British State has a hereditary

head, Indian State is headed by directly

elected head.

Select the correct answer using the code

given below.

(a) 1 only

(b) 2 only

(c) Both 1 and 2

(d) Neither 1 nor 2

Page 20: xamlibrary.files.wordpress.com · 2018-09-01 · 5 ©Vision IAS 17. The Indian Councils Act of 1909 provided for which of the following provisions? 1. Non-official majority in Central

1 www.visionias.in ©Vision IAS

VISIONIAS www.visionias.in

ANSWERS & EXPLANATION

GENERAL STUIDES (P) TEST – 2651 (2019)

Q 1.C

Statement 1 is correct: The purpose of Article 32 is to provide a guaranteed, effective, expeditious,

inexpensive and summary remedy for the protection of the fundamental rights. Only the Fundamental

Rights guaranteed by the Constitution can be enforced under Article 32 and not any other right like non-

fundamental constitutional rights, statutory rights, customary rights and so on. In other words, the

Supreme Court, under Article 32, cannot determine a question that does not involve Fundamental Rights.

Article 32 cannot be invoked simply to determine the constitutionality of an executive order or a

legislation unless it directly infringes any of the fundamental rights.

Statement 2 is not correct: In case of the enforcement of Fundamental Rights, the jurisdiction of the

Supreme Court is original but not exclusive. It is concurrent with the jurisdiction of the high court under

Article 226.

Statement 3 is correct: The Supreme Court has ruled that Article 32 is a basic feature of the Constitution.

Hence, it cannot be abridged or taken away even by way of an amendment to the Constitution.

Q 2.B

Option 1 is correct: The Indian Councils Act of 1861 empowered the Viceroy to issue ordinances,

without the concurrence of the legislative council, during an emergency. The life of such an ordinance

was six months.

Option 2 is correct: The Indian Councils Act of 1861 gave a recognition to the ‗portfolio‘ system,

introduced by Lord Canning in 1859. Under this, a member of the Viceroy‘s council was made in-charge

of one or more departments of the government and was authorised to issue final orders on behalf of the

council on matters of his department(s).

Option 3 is not correct: Government of India Act of 1919 introduced bicameralism in India. The Indian

Legislative Council was replaced by a bicameral legislature consisting of an Upper House (Council of

State) and a Lower House (Legislative Assembly).

Q 3.D

The Central Government shall appoint the Director of CBI on the recommendation of a three-member

committee consisting of the Prime Minister as Chairperson, the Leader of Opposition in the Lok

Sabha and the Chief Justice of India or Judge of the Supreme Court nominated by him. Hence, only

options 1 and 4 are correct.

Q 4.C

Right against Exploitation includes Article 23 and 24 of the Constitution.

Article 23 prohibits traffic in human beings, begar (forced labour) and other similar forms of forced

labour.

Article 24 prohibits the employment of children below the age of 14 years in any factory, mine or other

hazardous activities like construction work or railway.

Protection against arrest and detention is provided under Article 22 and is under Right to Freedom

(Art.19-22)

Page 21: xamlibrary.files.wordpress.com · 2018-09-01 · 5 ©Vision IAS 17. The Indian Councils Act of 1909 provided for which of the following provisions? 1. Non-official majority in Central

2 www.visionias.in ©Vision IAS

Q 5.A

o Democracy as a form of government only ensures that people take their own decisions. This does not

guarantee that their decisions will be good. People can make mistakes. Involving the people in these

decisions does lead to delays in decision making. It is also true that democracy leads to frequent

changes in leadership. Sometimes this can set back big decisions and affect the government‘s efficiency.

Democracy may lead to corruption for it is based on electoral competition due to more expenditure

in elections, criminalisation of politics, etc.

Democracy is based on consultation and discussion. A democratic decision always involves many

persons, discussions and meetings. When a number of people put their heads together, they are able to

point out possible mistakes in any decision. This takes time. But there is a big advantage in taking time

over important decisions. This reduces the chances of rash or irresponsible decisions. Thus democracy

improves the quality of decision-making.

Q 6.D

Statement 1 is not correct: In Berubari Union Case (1960), the Supreme Court specifically opined that

the Preamble is 'not' a part of the Constitution. However, in Kesavananda Bharti Case (1973), the

Supreme Court reversed its opinion and held that Preamble is an integral part of the Constitution.

Statement 2 is not correct: In the same Kesavananda Bharti Case (1973), the Supreme Court held that

the Preamble can be amended, subject to the condition that no amendment is done to 'basic structure'. The

Constitution has been amended once till date- 42nd Constitutional Amendment of 1976 which added three

new words- Socialist, Secular and Integrity. This amendment was held to be valid by the Supreme Court.

Q 7.D

The National Commission for Scheduled Tribes was established under Article 338 of the Constitution

with the objective of monitoring the safeguards provided for the STs under the Constitution or other laws.

The functions of the National Commission for STs include:

o To inquire into specific complaints with respect to the deprivation of rights and safeguards of the STs.

o To suggest measures to be taken over conferring ownership rights in respect of minor forest produce

to STs living in forest areas.

o To suggest measures to be taken to ensure full implementation of the Provisions of Panchayats

(Extension to the Scheduled Areas) Act, 1996.

Hence, all the statements are correct.

Q 8.D

The doctrine of Basic structure of the Constitution was laid down by the Supreme Court in the case of

Kesavananda Bharati v/s State of Kerala (1973). Based on various judgements of the Court, the following

are some of the elements included in the Basic structure: Supremacy of the Constitution; Secular

character of the Constitution; Sovereign, democratic and republic nature of polity; Separation of

powers between the legislature, judiciary and executive; Unity and Integrity of the nation; Welfare

state; Rule of Law; Parliamentary system; Effective access to justice; Principle of reasonableness etc

Socialist state is not included in the Basic structure.

Q 9.C

The Constitution (Article 4) itself declares that laws made for admission or establishment of new states

that are not part of the Union of India (under Article 2) and formation of new states and alteration of

areas, boundaries or names of existing states (under Articles 3) are not to be considered as amendments of

the Constitution under Article 368. This means that such laws can be passed by a simple majority of the

members present and voting and by the ordinary legislative process.

Q 10.C

Statement 1 is correct: One of the many reasons for naming Fundamental rights as so is because they are

essential for the all-round development i.e. material, intellectual, moral and spiritual of the individuals.

Statement 2 is not correct: Fundamental rights are provided under part III of Indian Constitution.

However, these are not the only rights provided by the constitution. Rights outside part III include -:

Page 22: xamlibrary.files.wordpress.com · 2018-09-01 · 5 ©Vision IAS 17. The Indian Councils Act of 1909 provided for which of the following provisions? 1. Non-official majority in Central

3 www.visionias.in ©Vision IAS

o Art. 265 - No tax without Law

o Art. 300A - Property Rights

o Art.301 - Free Trade, Commerce & Intercourse

o Art. 326 - Elections based on Adult Suffrage.

Statement 3 is not correct: They are not sacrosanct or permanent. The Parliament can curtail or repeal

them but only by a constitutional amendment act and not by an ordinary act. Moreover, this can be done

without affecting the ‗basic structure‘ of the Constitution..

Q 11.B

The phrase ‘Directive Principles of State Policy’ denotes the ideals that the State should keep in mind

while formulating policies and enacting laws. These are the constitutional instructions, or

recommendations to the State in legislative, executive and administrative matters. The term ‗State‘

here includes the legislative and executive organs of the central and state governments, all local

authorities and all other public authorities in the country.

The Directive Principles are non-justiciable in nature, that is, they are not legally enforceable by the courts

for their violation. Therefore, the government (Central, state and local) cannot be compelled to implement

them. Nevertheless, the Constitution (Article 37) itself says that these principles are fundamental in the

governance of the country and it shall be the duty of the State to apply these principles in making laws.

The Directive Principles, though non-justiciable in nature, help the courts in examining and determining

the constitutional validity of a law. The Supreme Court has ruled many a time that in determining the

constitutionality of any law, if a court finds that the law in question seeks to give effect to a Directive

Principle, it may consider such law to be ‗reasonable‘ in relation to Article 14 (equality before law) or

Article 19 (six freedoms) and thus save such law from unconstitutionality.

Q 12.A

Article 18 abolishes titles and makes four provisions in that regard:

o It prohibits the state from conferring any title (except a military or academic distinction) on any

body, whether a citizen or a foreigner. Hence, hereditary titles of nobility like Maharaja, Raj

Bahadur, Rai Bahadur, Rai Saheb, Dewan Bahadur, etc, which were conferred by colonial

States are banned by Article 18 as these are against the principle of equal status of all.

o It prohibits a citizen of India from accepting any title from any foreign state.

o A foreigner holding any office of profit or trust under the state cannot accept any title from any

foreign state without the consent of the president.

o No citizen or foreigner holding any office of profit or trust under the State is to accept any present,

emolument or office from or under any foreign State without the consent of the president.

In 1996, the Supreme Court upheld the constitutional validity of the meritorious awards like

National Awards—Bharat Ratna, Padma Vibhushan, Padma Bhushan and Padma Sri. It ruled that

these awards do not amount to ‘titles’ within the meaning of Article 18 that prohibits only

hereditary titles of nobility. Therefore, they are not violative of Article 18 as the theory of equality

does not mandate that merit should not be recognised. However, it also ruled that they should not be used

as suffixes or prefixes to the names of awardees. Otherwise, they should forfeit the awards.

Q 13.C

Article 359 authorises the President to suspend the right to move any court for the enforcement of

Fundamental Rights during a National Emergency. This means that under Article 359, the

Fundamental Rights as such are not suspended, but only their enforcement. The said rights are

theoretically alive but the right to seek remedy is suspended. The suspension of enforcement relates to

only those Fundamental Rights that are specified in the Presidential Order. Further, the suspension

could be for the period during the operation of emergency or for a shorter period as mentioned in the

order, and the suspension order may extend to the whole or any part of the country. It should be laid

before each House of Parliament for approval. Hence, only option (c) is correct.

The 44th Amendment Act of 1978 restricted the scope of Article 359 in two ways. Firstly, the

President cannot suspend the right to move the Court for the enforcement of fundamental rights

guaranteed by Articles 20 to 21. Secondly, only those laws which are related with the emergency are

Page 23: xamlibrary.files.wordpress.com · 2018-09-01 · 5 ©Vision IAS 17. The Indian Councils Act of 1909 provided for which of the following provisions? 1. Non-official majority in Central

4 www.visionias.in ©Vision IAS

protected from being challenged and not other laws and the executive action taken only under such a law,

is protected.

Article 359 operates in case of both when National Emergency is declared on grounds of war or external

aggression as well as on the ground of armed rebellion.

Article 359 may extend to the entire country or a part of it.

Q 14.D

In 1954, the States Reorganisation Commission was set up and it recommended the creation of

linguistic States, at least for the major linguistic groups. In 1956, reorganisation of some States took place.

This saw the beginning of the creation of linguistic States and the process is still continuing.

Punjab and Haryana were separated from each other in 1966.

36th Constitutional Amendment Act (1975) was enacted to make Sikkim a full-fledged state of the

Indian Union (the 22nd state).

In the 1990s, some of the larger States were further divided both to meet the demands for a separate State

as well as to meet the need for greater administrative efficiency. Thus Bihar, UP, and Madhya Pradesh

were divided to create three new States through 84th Constitutional Amendment Act, 2000. They are:

Jharkhand, Uttaranchal(now Uttarakhand) and Chhattisgarh respectively.

Q 15.A

Freedom of Religion is a Fundamental Right provided under Articles 25-28 of the Constitution. Freedom

of religion also includes the freedom of conscience. This means that a person may choose any religion or

may choose not to follow any religion. Hence, statement 1 is correct.

Freedom of Religion includes the right to not just practice one‘s religion but also to propagate it. This

includes persuading people to join one‘s religion and willful conversion from one religion to another. The

Constitution bars forceful conversions and conversions done by inducements. Hence, statement 2 is not

correct.

Freedom of Religion is available to not just Indian citizens but also to foreign nationals residing within

Indian territory. Hence, statement 3 is not correct.

Q 16.A

The Constitution of India is the lengthiest of all the written constitutions of the world. It is a very

comprehensive, elaborate and detailed document. The Constitution contains not only the fundamental

principles of governance but also detailed administrative provisions. Four factors have contributed to the

elephantine size of our Constitution. They are:

o Geographical factors, that is, the vastness of the country and its diversity.

o Historical factors, e.g., the influence of the Government of India Act of 1935, which was bulky.

o Single Constitution for both the Centre and the states except Jammu and Kashmir.

o Dominance of legal luminaries in the Constituent Assembly led to comprehensive coverage of

legal provisions in the Constitution.

Q 17.C

Features of the Act of Indian Councils Act of 1909 (Morley-Minto Reforms):

o It considerably increased the size of the legislative councils, both Central and provincial. The number

of members in the Central Legislative Council was raised from 16 to 60. The number of members in

the provincial legislative councils was not uniform.

o It retained official majority in the Central Legislative Council but allowed the provincial legislative

councils to have non-official majority. Hence, statement 1 is not correct.

o It enlarged the deliberative functions of the legislative councils at both the levels. For example,

members were allowed to ask supplementary questions, move resolutions on the budget, and so on.

o It provided (for the first time) for the association of Indians with the executive Councils of the

Viceroy and Governors. Satyendra Prasad Sinha became the first Indian to join the Viceroy‘s

Executive Council. He was appointed as the law member. Hence, statement 2 is correct.

o It introduced a system of communal representation for Muslims by accepting the concept of ‗separate

electorate‘. Under this, the Muslim members were to be elected only by Muslim voters. It was

Page 24: xamlibrary.files.wordpress.com · 2018-09-01 · 5 ©Vision IAS 17. The Indian Councils Act of 1909 provided for which of the following provisions? 1. Non-official majority in Central

5 www.visionias.in ©Vision IAS

extended to Sikhs, Indian Christians, Anglo-Indians and Europeans by Government of India Act of

1919. Hence, statement 3 is not correct.

o It also provided for the separate representation of presidency corporations, chambers of commerce,

universities and zamindars.

Q 18.B

Cases related to dispute/conflict between Fundamental Right and Directive Principles of State Policy

(DPSPs) -:

o Champakam Dorairajan Case, 1951 - In this case, Supreme Court ruled that in case of any conflict

between FRs and DPSPs, FRs would prevail. It declared that DPSPs have to conform to and run as

subsidiary to the fundamental right. However, it also held that FRs could be amended by the

Parliament through constitutional amendment acts. This led to First, Fourth and Seventeenth

Amendment Act to implement some of the DPSPs.

o Golak Nath case, 1967 - In this case, Supreme Court held that Parliament can‘t take away or abridge

any of the Fundamental Rights, which are ‗sacrosanct‘ in nature. Hence, court held that Fundamental

Rights can‘t be amended for the implementation of DPSPs. This led to enactment of 24th Amendment

Act and 25 Amendment Act which inserted a new article 31C.

o Kesavanand Bharti case, 1973 - In this case, Supreme Court declared the second provision of Article

31C as unconstitutional and invalid on the ground that judical review is a basic feature of

Constitution. This led to enactment of 42nd Amendment Act which gave legal primacy and

supremacy to the DPSPs over FRs conferred by article 14, 19 and 21.

o Minerva Mills case, 1980- In this case, Supreme Court held primacy of DPSPs over FRs as

unconstitutional and invalid. It led to subordination of DPSPs over FRs. However, FRs conferred by

Article 14 and 19 were accepted as subordinate to the DPSPs specified under Article 39(b) and (c).

The present position is that the FRs enjoy supremacy over DPSPs. However, Parliament can amend the

FRs for implementing the DPSP, so long as the amendment doesn‘t destroy of the basic feature of the

consitution.

Maneka Gandhi case, 1980 - It deals with right guaranteed under Article 21 of Constitution. Prior to this,

there was ‗procedure prescribed by law ‗which was replaced by ‗due process of law‘ with respect to

article 21 of Constitution.

IR Coelho case in 2007—popularly known as the Ninth Schedule case—the Supreme Court took this

further and argued that if the purpose of inserting a law into the Ninth Schedule was to undo a judgment

of the Supreme Court, this could be examined by the courts. Also,the Supreme Court held that the laws

placed under IX schedule after Kesavananda Bharati judgment (24th April 1973 ) cannot be exempt from

Judicial review.

Q 19.C

Article 360 empowers the President to proclaim a Financial Emergency if he is satisfied that a

situation has arisen due to which the financial stability or credit of India or any part of its territory

is threatened. The 38th Amendment Act of 1975 made the satisfaction of the President in declaring a

Financial Emergency final and conclusive and not questionable in any court on any ground. But, this

provision was subsequently deleted by the 44th Amendment Act of 1978 implying that the satisfaction of

the President is not beyond judicial review.

Hence, option (c) is correct.

Q 20.C

According to Article 51 A, it shall be the duty of every citizen of India:

o to abide by the Constitution and respect its ideals and institutions, the National Flag and the National

Anthem;

o to cherish and follow the noble ideals that inspired the national struggle for freedom;

o to uphold and protect the sovereignty, unity and integrity of India;

o to defend the country and render national service when called upon to do so;

o to promote harmony and the spirit of common brotherhood amongst all the people of India

transcending religious, linguistic and regional or sectional diversities and to renounce practices

derogatory to the dignity of women;

Page 25: xamlibrary.files.wordpress.com · 2018-09-01 · 5 ©Vision IAS 17. The Indian Councils Act of 1909 provided for which of the following provisions? 1. Non-official majority in Central

6 www.visionias.in ©Vision IAS

o to value and preserve the rich heritage of the country‘s composite culture;

o to protect and improve the natural environment including forests, lakes, rivers and wildlife and to

have compassion for living creatures; (h) to develop scientific temper, humanism and the spirit of

inquiry and reform;

o to safeguard public property and to abjure violence;

o to strive towards excellence in all spheres of individual and collective activity so that the nation

constantly rises to higher levels of endeavour and achievement; and

o to provide opportunities for education to his child or ward between the age of six and fourteen years.

This duty was added by the 86th Constitutional Amendment Act, 2002.

Q 21.A

According to the Constitution, the following four categories of persons became the citizens of India at its

commencement i.e., on 26 January, 1950:

o Article 5 says that "A person who had his domicile in India and also fulfilled any one of the three

conditions, viz., if he was born in India; or if either of his parents was born in India; or if he has been

ordinarily resident in India for five years immediately before the commencement of the

Constitution, became a citizen of India. Hence, only option (a) is not correct.

o Article 6 says that "A person who migrated to India from Pakistan became an Indian citizen if he or

either of his parents or any of his grandparents was born in undivided India and also fulfilled

any one of the two conditions viz., in case he migrated to India before July 19, 1948, he had been

ordinarily resident in India since the date of his migration; or in case he migrated to India on or

after July 19, 1948, he had been registered as a citizen of India. But, a person could be so

registered only if he had been resident in India for six months preceding the date of his application for

registration.

o Article 7 says that "A person who migrated to Pakistan from India after March 1, 1947, but later

returned to India for resettlement could become an Indian citizen. For this, he had to be resident

in India for six months preceding the date of his application for registration.

o Article 8 says that "A person who, or any of whose parents or grandparents, was born in

undivided India but who is ordinarily residing outside India shall become an Indian citizen if he

has been registered as a citizen of India by the diplomatic or consular representative of India in the

country of his residence, whether before or after the commencement of the Constitution. Thus, this

provision covers the overseas Indians who may want to acquire Indian citizenship.

Q 22.B

Prohibition: Literally, it means ‗to forbid‘. It is issued by a higher court to a lower court or tribunal to

prevent the latter from exceeding its jurisdiction or usurping a jurisdiction that it does not possess. Thus,

unlike mandamus that directs activity, the prohibition directs inactivity.

Certiorari: In the literal sense, it means ‗to be certified‘ or ‗to be informed‘. It is issued by a higher court

to a lower court or tribunal either to transfer a case pending with the latter to itself or to quash the order of

the latter in a case. It is issued on the grounds of excess of jurisdiction or lack of jurisdiction or error of

law. Thus, unlike prohibition, which is only preventive, certiorari is both preventive as well as curative.

Mandamus: It literally means ‗we command‘. It is a command issued by the court to a public official

asking him to perform his official duties that he has failed or refused to perform. It can also be issued

against any public body, a corporation, an inferior court, a tribunal or government for the same purpose.

Quo-Warranto: In the literal sense, it means ‗by what authority or warrant‘. It is issued by the court to

enquire into the legality of claim of a person to a public office. Hence, it prevents illegal usurpation of

public office by a person.

Q 23.B

The Central Vigilance Commission (CVC) is the main agency for preventing corruption in the Central

government. It was established in 1964 by an executive resolution of the Central government. Its

establishment was recommended by the Santhanam Committee on Prevention of Corruption (1962–64).

Thus, originally the CVC was neither a constitutional body nor a statutory body. Later, in 2003, the

Parliament enacted a law conferring statutory status on the CVC. Hence, statement 1 is not correct.

Page 26: xamlibrary.files.wordpress.com · 2018-09-01 · 5 ©Vision IAS 17. The Indian Councils Act of 1909 provided for which of the following provisions? 1. Non-official majority in Central

7 www.visionias.in ©Vision IAS

The CVC is a multi-member body consisting of a Central Vigilance Commissioner (chairperson) and not

more than two vigilance commissioners. They are appointed by the President by warrant under his

hand and seal on the recommendation of a three-member committee consisting of the prime minister as

its head, the Union minister of home affairs and the Leader of the Opposition in the Lok Sabha. They hold

office for a term of four years or until they attain the age of sixty five years, whichever is earlier. After

their tenure, they are not eligible for further employment under the Central or a state government. Hence,

statement 2 is correct.

Q 24.D

Statement 1 is not correct: State Reorganisation Commission (Fazl Ali Commission) was setup under

the chairmanship of Fazl Ali in 1953. It submitted its report in September 1955 and broadly accepted

language as the basis of reorganisation of states. But, it rejected the theory of 'one language-one state'. Its

view was that unity of India should be regarded as primary consideration in any redrawing of country's

political units.

Statement 2 is not correct: Andhra Pradesh was the first state to be formed on linguistic basis in 1953. It

was created by separating Telugu speaking areas from the Madras state. While Gujarat was the first state

to be created on linguistic basis after the States Reorganisation Act of 1956. Gujarat was created after

separating Gujarati-speaking areas from Marathi-speaking areas of Maharashtra in 1960.

Q 25.C

Regulating Act of 1773 is of great constitutional importance as

o it was the first step taken by the British Government to control and regulate the affairs of the East

India Company in India.

o it recognised, for the first time, the political and administrative functions of the Company. Hence,

statement 1 is correct.

o it laid the foundations of central administration in India. Hence, statement 2 is correct.

Features of the Act

o It designated the Governor of Bengal as the ‗Governor-General of Bengal‘ and created an Executive

Council of four members to assist him. The first such Governor-General was Lord Warren Hastings.

o It made the governors of Bombay and Madras presidencies subordinate to the governor-general of

Bengal, unlike earlier, when the three presidencies were independent of one another.

o It provided for the establishment of a Supreme Court at Calcutta (not Delhi) comprising one chief

justice and three other judges.

o It prohibited the servants of the Company from engaging in any private trade or accepting presents or

bribes from the ‗natives‘.

o It strengthened the control of the British Government over the Company by requiring the Court of

Directors (governing body of the Company) to report on its revenue, civil, and military affairs in

India.

Q 26.B

Statement 1 is not correct: During a National emergency (Article 352), the executive power of the

Centre extends to directing any state regarding the manner in which its executive power is to be exercised.

In normal times, the Centre can give executive directions to a state only on certain specified matters.

However, during a national emergency, the Centre becomes entitled to give executive directions to a

state on ‘any’ matter. Thus, the state governments are brought under the complete control of the

Centre, though they are not suspended.

Statement 2 is correct: The six rights guaranteed by Article 19 can be suspended only when emergency

is declared on the grounds of war or external aggression (i.e., external emergency) and not on the ground

of armed rebellion. Hence, they constitute a limitation on fundamental rights under Art.19.

Q 27.A

The 44th Amendment Act of 1978 abolished the right to property as a Fundamental Right by repealing

Article 19(1)(f) and Article 31 from Part III. Instead, the Act inserted a new Article 300A in Part XII

under the heading ‗Right to Property‘. It provides that no person shall be deprived of his property except

by authority of law.

Page 27: xamlibrary.files.wordpress.com · 2018-09-01 · 5 ©Vision IAS 17. The Indian Councils Act of 1909 provided for which of the following provisions? 1. Non-official majority in Central

8 www.visionias.in ©Vision IAS

Thus, Right to property now no longer remains a fundamental right. It is not a part of the basic

structure of the Constitution.The right to property as a legal right (as distinct from the Fundamental

Rights) has the following implications:

o It can be regulated i.e., curtailed, abridged or modified without constitutional amendment by an

ordinary law of the Parliament.

o It protects private property against executive action but not against legislative action.

o In case of violation, the aggrieved person cannot directly move the Supreme Court under

Article 32 (right to constitutional remedies including writs) for its enforcement. He can move to

the High Court under Article 226.

o No guaranteed right to compensation in case of acquisition or requisition of the private property by

the state.

Q 28.A

The Commission has to investigate all matters relating to the constitutional and other legal

safeguards for the SC's, OBCs and the Anglo-Indian Community and report to the President upon their

working. Hence, statement 1 is correct.

The National Commission for Scheduled Castes presents an annual report to the President. It can

also submit a report as and when it thinks necessary. The President places all such reports before the

Parliament, along with a memorandum explaining the action taken on the recommendations made by the

Commission. The memorandum should also contain the reasons for the non-acceptance of any of such

recommendations. The President also forwards any report of the Commission pertaining to a state

government to the state Governor. The Governor places it before the state legislature, along with a

memorandum explaining the action taken on the recommendations of the Commission. The memorandum

should also contain the reasons for the non-acceptance of any of such recommendations. Hence,

statement 2 is not correct.

Q 29.D

Article 16 provides for equality of opportunity for all citizens in matters of employment or appointment to

any office under the State. No citizen can be discriminated against or be ineligible for any employment or

office under the State on grounds of only religion, race, caste, sex, descent, place of birth or residence.

There are three exceptions to this general rule of equality of opportunity in public employment:

o Parliament can prescribe residence as a condition for certain employment or appointment in a state or

union territory or local authority or other authority. As the Public Employment (Requirement as to

Residence) Act of 1957 expired in 1974, there is no such provision for any state except Andhra

Pradesh.

o The State can provide for reservation of appointments or posts in favour of any backward class that is

not adequately represented in the state services.

o A law can provide that the incumbent of an office related to religious or denominational institution or

a member of its governing body should belong to the particular religion or denomination.

Q 30.A

The following were included in the Objectives Resolution as passed by the Constituent Assembly- India is

an independent, sovereign, republic; All powers and authority of independent India and its constitution

shall flow from the people.

Summary of Objectives resolution-

o India is an independent, sovereign, republic;

o India shall be a Union of erstwhile British Indian territories, Indian States, and other parts outside

British India and Indian States as are willing to be a part of the Union;

o Territories forming the Union shall be autonomous units and exercise all powers and functions of the

Government and administration, except those assigned to or vested in the Union;

o All powers and authority of sovereign and independent India and its constitution shall flow from the

people;

Page 28: xamlibrary.files.wordpress.com · 2018-09-01 · 5 ©Vision IAS 17. The Indian Councils Act of 1909 provided for which of the following provisions? 1. Non-official majority in Central

9 www.visionias.in ©Vision IAS

o All people of India shall be guaranteed and secured social, economic and political justice; equality of

status and opportunities and equality before law; and fundamental freedoms - of speech, expression,

belief, faith, worship, vocation, association and action - subject to law and public morality;

o The minorities, backward and tribal areas, depressed and other backward classes shall be

provided adequate safeguards;

o The territorial integrity of the Republic and its sovereign rights on land, sea and air shall be

maintained according to justice and law of civilized nations;

o The land would make full and willing contribution to the promotion of world peace and welfare

of mankind.

Q 31.C

Statement 1 is correct: The Constitution (Article 165) has provided for the office of the Advocate

General for the states. He is the highest law officer in the state. The Advocate General is appointed by the

Governor.

Statement 2 is not correct: Eligibilty conditions for Advocate General does not include minimum age

limits. He must be a person who is qualified to be appointed a judge of a high court. In other words, he

must be a citizen of India and must have held a judicial office for ten years or been an advocate of a high

court for ten years.

Statement 3 is correct: The remuneration of the Advocate General is not fixed by the Constitution. He

receives such remuneration as the Governor may determine.

Q 32.D

Federal Government Unitary Government

1. Dual Government (that is, national

government and regional government)

Single government, that is, the national government

which may create regional governments

2. Written Constitution Constitution may be written (France) or unwritten

(Britain)

3. Division of powers between the

national and regional government

No division of powers. All powers are vested in the

national government

4. Supremacy of the Constitution Constitution may be supreme (Japan) or may not be

supreme (Britain)

5. Rigid Constitution Constitution may be rigid (France) or flexible (Britain)

6. Independent judiciary Judiciary may be independent or may not be

independent

7. Bicameral legislature Legislature may be bicameral (Britain) or uni-cameral

(China)

Q 33.A

Article 22 grants protection to persons who are arrested or detained. Detention is of two types, namely,

punitive and preventive. Punitive detention is to punish a person for an offence committed by him after

trial and conviction in a court. Preventive detention, on the other hand, means detention of a person

without trial and conviction by a court. Its purpose is not to punish a person for a past offence but to

prevent him from committing an offence in the near future.

The Article 22 has two parts—the first part deals with the cases of ordinary law and the second part deals

with the cases of preventive detention law.

The second part of Article 22 grants protection to persons who are arrested or detained under a preventive

detention law. This protection is available to both citizens as well as aliens and includes the following:

o The detention of a person cannot exceed three months unless an advisory board reports

sufficient cause for extended detention. The board is to consist of judges of a high court.

o The grounds of detention should be communicated to the detenu. However, the facts considered

to be against the public interest need not be disclosed.

o The detenu should be afforded an opportunity to make a representation against the detention

order.

Page 29: xamlibrary.files.wordpress.com · 2018-09-01 · 5 ©Vision IAS 17. The Indian Councils Act of 1909 provided for which of the following provisions? 1. Non-official majority in Central

10 www.visionias.in ©Vision IAS

o A person has the right to be released after 24 hours (unless the magistrate authorises further

detention) only if he is arrested under ordinary law. It is not applicable in case of preventive

detention law.

Hence, only statement 3 is correct.

Q 34.B

A democracy must ensure that individuals have certain rights and that the government will always

recognise these rights. Therefore it is often a practice in most democratic countries to list the rights of the

citizens in the Constitution itself. Such a list of rights mentioned and protected by the Constitution is

called the ‘bill of rights’. A bill of rights prohibits government from acting against the rights of the

individuals and ensures a remedy in case there is violation of these rights. In the Indian context, the

Fundamental Rights mentioned in Part III of the Constitution form the 'bill of rights'.

Q 35.C

The Lokpal shall consist of a Chairperson with a maximum of 8 members of which 50% shall be judicial

members. 50% of the members of the Lokpal shall come from amongst the SCs, the STs, the OBCs,

minorities and women.

The Lokpal will have the power of superintendence and direction over any investigating agency, including

the CBI, for cases referred to them by the Lokpal. A High-Powered Committee chaired by the Prime

Minister will recommend the selection of the Director of CBI.

Hence, both the statements are correct.

Q 36.D

―There shall be a Council of Ministers with the Prime Minister at the head to aid and advice the President

who shall act in accordance with such advice‖, was added as Art 74(1) by the 42nd Constitutional

Amendment Act.

The minimum age for voting was reduced to 18 years by 61st Constitutional Amendment Act.

―The State can make special provisions for the advancement of socially and educationally backward

classes and citizens of Scheduled castes and Scheduled tribes‖, was added as Art 15(4) by the First

Constitutional Amendment Act.

Hence, all of the above provisions were added through Constitutional Amendments.

Q 37.A

The Constitution of India (Article 148) provides for an independent office of the Comptroller and Auditor

General of India (CAG). He is the head of the Indian Audit and Accounts Department. He is the

guardian of the public purse and controls the entire financial system of the country at both the levels—the

Centre and the state. Hence, statement 1 is correct.

He holds office for a period of six years or upto the age of 65 years, whichever is earlier. Hence,

statement 2 is not correct.

No minister can represent the CAG in Parliament (both Houses) and no minister can be called upon to

take any responsibility for any actions done by him. Hence, statement 3 is not correct.

Q 38.B

Indian Independence Act of 1947 ended the British rule in India and declared India as an independent and

sovereign state from August 15, 1947.

It provided for the partition of India and creation of two independent dominions of India and Pakistan

with the right to secede from the British Commonwealth.

It abolished the office of the secretary of state for India and transferred his functions to the

secretary of state for Commonwealth Affairs.

Q 39.A

Gandhian Principles: These principles are based on Gandhian ideology. They represent the programme

of reconstruction enunciated by Gandhi during the national movement. In order to fulfil the dreams of

Gandhi, some of his ideas were included as Directive Principles. They require the State:

Page 30: xamlibrary.files.wordpress.com · 2018-09-01 · 5 ©Vision IAS 17. The Indian Councils Act of 1909 provided for which of the following provisions? 1. Non-official majority in Central

11 www.visionias.in ©Vision IAS

o To organise village panchayats and endow them with necessary powers and authority to enable them

to function as units of self-government (Article 40).

o To promote cottage industries on an individual or co-operation basis in rural areas (Article 43).

o To promote voluntary formation, autonomous functioning, democratic control and professional

management of co-operative societies. (Article 43B).

o To promote the educational and economic interests of SCs, STs, and other weaker sections of the

society and to protect them from social injustice and exploitation (Article 46).

o To prohibit the consumption of intoxicating drinks and drugs which are injurious to health (Article

47).

o To prohibit the slaughter of cows, calves and other milch and draught cattle and to improve their

breeds (Article 48).

To organise agriculture and animal husbandry on modern and scientific lines (Article 48) is a

principle based on the ideology of liberalism.

To raise the level of nutrition and the standard of living of people and to improve public health

(Article 47) reflect the ideology of socialism.

Q 40.A

The 97th Constitutional Amendment Act of 2011 gave a constitutional status and protection to co-

operative societies. In this context, it made the following three changes in the constitution:

o It made the right to form co-operative societies a fundamental right (Article 19).

o It included a new Directive Principle of State Policy on promotion of cooperative societies (Article

43-B).

o It added a new Part IX-B in the Constitution which is entitled ―The Cooperative Societies‖ (Articles

243-ZH to 243-ZT).

Q 41.A

Originally, the Constitution of India did not make any provision with respect to the Special Officer for

Linguistic Minorities. Later, the States Reorganisation Commission (1953-55) made a recommendation in

this regard. Accordingly, the Seventh Constitutional Amendment Act of 1956 inserted a new Article

350-B in Part XVII of the Constitution.Hence, statement 1 is correct. This article contains the

following provisions:

o There should be a Special Officer for Linguistic Minorities. He is to be appointed by the President of

India.

o It would be the duty of the Special Officer to investigate all matters only relating to the

safeguards provided for linguistic minorities under the Constitution.

o He would report to the President upon those matters at such intervals as the President may direct. The

President should place all such reports before each House of Parliament and send to the governments

of the states concerned. Hence, statement 2 is not correct.

Q 42.B

The Preamble of the Constitution defines Indian Republic as Sovereign, Socialist, Secular and

Democratic. Of these, Sovereign and Democratic were part of the Preamble adopted by the Constituent

Assembly.

Socialist and Secular were added to the Preamble by the 42nd Constitutional Amendment Act of

1976. Also, the word 'integrity' was added by the same act.

Q 43.B

The National Human Rights Commission (NHRC) is composed of a former chief justice of the Supreme

Court of India, a former judge of the Supreme Court, a former chief justice of a High Court and two other

members who have knowledge and practical experience in matters relating to human rights.

The Commission‘s functions include inquiry at its own initiative(suo-moto) or on a petition presented to

it by a victim into complaint of violation of human rights; visit to jails to study the condition of the

inmates; undertaking and promoting research in the field of human rights, etc.

The Commission receives complaints in thousands every year. These relate to custodial death, custodial

rape, disappearances, police excesses, failure in taking action, indignity to women, etc. The Commission

Page 31: xamlibrary.files.wordpress.com · 2018-09-01 · 5 ©Vision IAS 17. The Indian Councils Act of 1909 provided for which of the following provisions? 1. Non-official majority in Central

12 www.visionias.in ©Vision IAS

does not have the power of prosecution. It can merely make recommendations to the government or

recommend to the courts to initiate proceedings based on the inquiry that it conducts.

Q 44.D

Some characteristics of the Fundamental Duties in the Indian Constitution are:

o Some of them are moral duties while others are civic duties. For instance, cherishing noble ideals of

freedom struggle is a moral precept and respecting the Constitution, National Flag and National

Anthem is a civic duty.

o They can be enforceable by law. Hence, the Parliament can provide for the imposition of appropriate

penalty or punishment for failure to fulfil any of them.

o Unlike some of the Fundamental Rights which extend to all persons whether citizens or foreigners,

the Fundamental Duties are confined to citizens only and do not extend to foreigners.

Q 45.C

The constitution of a country is a set of written rules that are accepted by all people living together in a

country. Constitution is the supreme law that determines the relationship among people living in a

territory (called citizens) and also the relationship between the people and government. A

constitution does many things:

o First, it generates a degree of trust and coordination that is necessary for different kind of people to

live together;

o Second, it specifies how the government will be constituted, who will have power to take which

decisions;

o Third, it lays down limits on the powers of the government and tells us what the rights of the

citizens are; and

o Fourth, it expresses the aspirations of the people about creating a good society.

Q 46.B

Statement 1 is not correct: The conditions of service and tenure of office of the election commissioners

and the regional commissioners shall be determined by the President. Under Article 324 of the

Constitution of India, the Election Commission of India is vested with the power of superintendence,

direction, and control of conducting the elections to the Lok Sabha and State Legislative Assemblies.

Statement 2 is not correct: In case of difference of opinion amongst the Chief election commissioner

and/or two other election commissioners, the matter is decided by the Commission by the majority.

Statement 3 is correct: He cannot be removed from his office except in the same manner and on the

same grounds as a judge of the Supreme Court. In other words, he can be removed by the president on the

basis of a resolution passed to that effect by both the Houses of Parliament with the special majority,

either on the ground of proved misbehavior or incapacity. Thus, he does not hold his office till the

pleasure of the president, though he is appointed by him.

Q 47.B

The Comptroller & Auditor General of India is provided with the security of tenure. He can be

removed by the President only in accordance with the procedure mentioned in the Constitution. Thus, he

does not hold his office till the pleasure of the president, though he is appointed by him. Also, he is not

eligible for further office, either under the Government of India or of any state, after he ceases to

hold his office. Hence, option (b) is correct.

The Attorney General is not a full-time counsel for the Government. He does not fall in the category

of government servants. Further, he is not debarred from private legal practice.

The chairman of UPSC (on ceasing to hold office) is not eligible for further employment in the

Government of India or a state. A member of UPSC (on ceasing to hold office) is eligible for

appointment as the chairman of UPSC or a State Public Service Commission (SPSC), but not for

any other employment in the Government of India or a state.

Q 48.D

The term ‗State‘ has been used in different provisions concerning the fundamental rights. Hence, Article

12 has defined the term for the purposes of Part III. According to it, the State includes the following:

Page 32: xamlibrary.files.wordpress.com · 2018-09-01 · 5 ©Vision IAS 17. The Indian Councils Act of 1909 provided for which of the following provisions? 1. Non-official majority in Central

13 www.visionias.in ©Vision IAS

o Government and Parliament of India, that is, executive and legislative organs of the Union

government.

o Government and legislature of states, that is, executive and legislative organs of state government.

o All local authorities, that is, municipalities, panchayats, district boards, improvement trusts, etc.

o All other authorities, that is, statutory or non-statutory authorities like LIC, ONGC, SAIL, etc.

Thus, State has been defined in a wider sense so as to include all its agencies. It is the actions of these

agencies that can be challenged in the courts as violating the Fundamental Rights.

According to the Supreme Court, even a private body or an agency working as an instrument of the State

falls within the meaning of the ‗State‘ under Article 12.

Hence, all the given options are correct.

Q 49.C

Both the statements are not correct.

The Constitution of India has opted for the British parliamentary System of Government rather than

American Presidential System of Government, both at the Centre and in the states. Articles 74 and 75 deal

with the parliamentary system at the Centre and Articles 163 and 164 in the states. Modern democratic

governments are classified into parliamentary and presidential on the basis of nature of relations between

the executive and the legislative organs of the government. The parliamentary system of government is

the one in which the executive is responsible to the legislature for its policies and acts. The presidential

system of government, on the other hand, is one in which the executive is not responsible to the legislature

for its policies and acts, and is constitutionally independent of the legislature in respect of its term of

office. The parliamentary system is based on the principle of cooperation and co-ordination

between the legislative and executive organs while the presidential system is based on the doctrine

of separation of powers between the two organs. The parliamentary system is also known as the

‗Westminster model of government, responsible government and cabinet government. The Constitution

establishes the parliamentary system not only at the Centre but also in the states. The features of

parliamentary government in India are:

o Presence of nominal and real executives;

o Majority party rule,

o Collective responsibility of the executive to the legislature,

o Membership of the ministers in the legislature,

o Leadership of the prime minister or the chief minister,

o Dissolution of the lower House (Lok Sabha or Assembly).

Q 50.B

The Protection of Human Rights Act of 1993 provides for the creation of not only the National Human

Rights Commission but also a State Human Rights Commission at the state level. Accordingly, twenty

three states have constituted the State Human Rights Commissions through Official Gazette Notifications.

A State Human Rights Commission can inquire into violation of human rights only in respect of

subjects mentioned in the State List (List-II) and the Concurrent List (List-III) of the Seventh

Schedule of the Constitution. However, if any such case is already being inquired into by the National

Human Rights Commission or any other Statutory Commission, then the State Human Rights

Commission does not inquire into that case.

Q 51.C

Statement 1 is correct: The term Federation has not been mentioned anywhere in the Consitution.

Statement 2 is correct: Article 1 describes India as a ‗Union of States‘ which implies two things: one,

Indian Federation is not the result of an agreement by the states; and two, no state has the right to secede

from the federation. According to Dr B R Ambedkar, the phrase ‗Union of States‘ has been preferred to

‗Federation of States‘ for two reasons: one, the Indian Federation is not the result of an agreement among

the states like the American Federation; and two, the states have no right to secede from the federation.

The federation is a Union because it is indestructible. The country is an integral whole and divided into

different states only for the convenience of administration.

Page 33: xamlibrary.files.wordpress.com · 2018-09-01 · 5 ©Vision IAS 17. The Indian Councils Act of 1909 provided for which of the following provisions? 1. Non-official majority in Central

14 www.visionias.in ©Vision IAS

Q 52.C

o Statement 1 is correct: Article 280 of the Constitution of India provides for a Finance Commission. It is

a quasi judicial body. The Finance Commission shall have all powers of civil court under Code of Civil

Procedure (1908) in matters of summoning & enforcing attendance and requisitioning any public

record from any court of office. Also Finance Commission shall be deemed to be a civil court for

purposes of sections 480 and 482 of the CrPC provided under Finance Commission Act

o It is constituted by the President of India every fifth year or at such earlier time as he considers necessary.

o Statement 2 is not correct: The Finance Commission consists of a chairman and four other members to

be appointed by the President. They hold office for such period as specified by the President in his order.

o Statement 3 is correct: They are eligible for reappointment.

Q 53.A

One thing that has had a long lasting effect on the evolution of the Indian Constitution is the theory of the

basic structure of the Constitution. The Judiciary advanced this theory in the famous case of

Kesavananda Bharati in 1973. This ruling has contributed to the evolution of the Constitution in the

following ways:

o It has set specific limits to Parliament‘s power to amend the Constitution. It says that no amendment

can violate the basic structure of the Constitution;

o It allows Parliament to amend any and all parts of the Constitution (within this limitation); and

o It places the Judiciary as the final authority in deciding if an amendment violates basic structure and

what constitutes the basic structure.

The theory of basic structure is an example of a living constitution. There is no mention of this theory in

the Constitution. It has emerged from judicial interpretation. Thus, the Judiciary and its interpretation

have practically amended the Constitution without a formal amendment. All living documents evolve in

this manner through debates, arguments, competition and practical politics.

Since 1973, the Court has, in many cases, elaborated upon this theory of basic structure and given

instances of what constitutes the basic structure of the Constitution of India. So, the basic structure

doctrine has further consolidated the balance between rigidity and flexibility: by saying that certain

parts cannot be amended, it has underlined the rigid nature while by allowing amendments to all others it

has underlined the flexible nature of the amending process.

Q 54.C

Cultural and educational rights are guaranteed under the Indian Constitution under Articles 29–30:

o Protection of language, script and culture of minorities (Article 29) - Any section of the citizens

residing in the territory of India or any part having a distinct language, script or culture of its own

shall have the right to conserve the same. Hence, option (c) is correct.

o Right of minorities to establish and administer educational institutions (Article 30).

The State shall provide free and compulsory education to all children of the age of six to fourteen years in

such a manner as the State may determine - Article 21-A

Article 28 - Freedom as to attendance at religious instruction or religious worship in certain education

institutions.

Article 351 - It shall be the duty of the Union to promote the spread of the Hindi language and to develop

it so that it may serve as a medium of expression for all the elements of the composite culture of India.

Q 55.D

o A number of provisions in the Constitution can be amended by a simple majority of the two Houses

of Parliament outside the scope of Article 368. These provisions include abolition or creation of

legislative councils in states, Citizenship—acquisition and termination etc.

o The majority of the provisions in the Constitution need to be amended by a special majority of the

Parliament, that is, a majority (that is, more than 50 per cent) of the total membership of each House and

a majority of two-thirds of the members of each House present and voting. The expression ‗total

membership‘ means the total number of members comprising the House irrespective of fact whether there

are vacancies or absentees. The provisions which can be amended by this way includes: (i)

Fundamental Rights; (ii) Directive Principles of State Policy

Page 34: xamlibrary.files.wordpress.com · 2018-09-01 · 5 ©Vision IAS 17. The Indian Councils Act of 1909 provided for which of the following provisions? 1. Non-official majority in Central

15 www.visionias.in ©Vision IAS

Q 56.C

The Constitution confers the following rights and privileges on the citizens of India (and denies the same

to aliens):

o Right against discrimination on grounds of religion, race, caste, sex or place of birth (Article 15).

o Right to equality of opportunity in the matter of public employment (Article 16).

o Right to freedom of speech and expression, assembly, association, movement, residence and

profession (Article 19). Hence, option 2 is correct.

o Cultural and educational rights (Articles 29 and 30).

o Right to vote in elections to the Lok Sabha and state legislative assembly.

o Right to contest for the membership of the Parliament and the state legislature. Hence, option 1 is

correct.

o Eligibility to hold certain public offices, that is, President of India, Vice-president of India, judges of

the Supreme Court and the high courts, governor of states, attorney general of India and advocate

general of states.

Rights to elementary education is conferred to citizens of India as well as aliens. Hence, option 3 is not

correct.

Q 57.A

Constitution

Amendment

Directive Principles added/changed

42nd, 1976 To secure opportunities for healthy development of children (Article 39)

To promote equal justice and to provide free legal aid to the poor (Article 39

A)

To take steps to secure the participation of workers in the management of

industries (Article 43 A)

To secure opportunities for healthy development of children (Article 39) To

protect and improve the environment and to safeguard forests and wild life

(Article 48 A).

44th, 1978 The State shall, in particular, strive to minimize the inequalities in income, and

endeavor to eliminate inequalities in status, facilities and opportunities, not

only amongst individuals but also amongst groups of people residing in different

areas or engaged in different vocations (Article 38(2))

97th, 2011 The State shall endeavour to promote voluntary formation, autonomous

functioning, democratic control and professional management of co-operative

societies. (Article 43-B)

86th, 2002 The State shall endeavour to provide early childhood care and education for all

children until they complete the age of six years. (The subject of article 45

changed)

Q 58.A

Statement 1 is correct: Article 14 provides for equality of law and equal protection of law. It lays down

the foundations of ‗Rule of Law‘ for Indian Constitution.

The concept of ‗equality before law‘ is an element of the concept of ‘Rule of Law’, propounded by

A.V. Dicey, the British jurist. His concept has the following three elements or aspects:(i) Absence of

arbitrary power, that is, no man can be punished except for a breach of law. (ii) Equality before the

law, that is, equal subjection of all citizens (rich or poor, high or low, official or non-official) to the

ordinary law of the land administered by the ordinary law courts. (iii) The primacy of the rights of the

individual, that is, the constitution is the result of the rights of the individual as defined and enforced by

the courts of law rather than the constitution being the source of the individual rights. The first and the

second elements are applicable to the Indian System and not the third one. In the Indian System, the

constitution is the source of the individual rights. The Supreme Court held that the ‘Rule of Law’ as

Page 35: xamlibrary.files.wordpress.com · 2018-09-01 · 5 ©Vision IAS 17. The Indian Councils Act of 1909 provided for which of the following provisions? 1. Non-official majority in Central

16 www.visionias.in ©Vision IAS

embodied in Article 14 is a ‘basic feature’ of the constitution. Hence, it cannot be destroyed even by an

amendment.

Statement 2 is not correct: Article 14 is not absolute and there are constitutional and other

exceptions to it. Some of them are -:

o The President or the Governor of a state is not answerable to any court for exercise and performance

of the powers and duties of his office.

o No criminal proceedings can be instituted against the President or a Governor during his term of

office.

o The President or a Governor cannot be arrested or imprisoned during his tenure of office. No civil

proceedings can be instituted against the President or a Governor during his tenure of office without

two months‘ written notice.

o Cases cannot be filed in the court against foreign sovereigns, ambassadors and the staff of the foreign

embassies.

o In times of war the enemy aliens cannot file cases in the Indian Courts nor can they claim equal

opportunities with other prisoners.

o The members of Parliament and the state legislatures enjoy a number of ‗privileges‘.

o According to the 44th Constitution (Amendment) Act, 1978, Parliament may, by law, set up

Administrative Tribunals for the adjudication of disputes relating to the service of the employees of

the government, or of any local or other authority.

Q 59.D

Third Schedule Forms of Oaths or Affirmations for:

o The Union ministers

o The candidates for election to the Parliament

o The members of Parliament

o The judges of the Supreme Court

o The Comptroller and Auditor-General of India

o The state ministers

o The candidates for election to the state legislature

o The members of the state legislature

o The judges of the High Courts

Fourth Schedule: Allocation of seats in the Rajya Sabha to the states and the union territories.

Second Schedule: Provisions relating to the emoluments, allowances, privileges of President, Governor,

etc.

Ninth Schedule: Acts and Regulations (originally 13 but presently 282) of the state legislatures dealing

with land reforms and abolition of the zamindari system and of the Parliament dealing with other matters.

This schedule was added by the 1st Amendment (1951) to protect the laws included in it from judicial

scrutiny on the ground of violation of fundamental rights. However, in 2007, the Supreme Court ruled that

the laws included in this schedule after April 24, 1973, are now open to judicial review.

Fifth Schedule: Administration and control of scheduled areas and scheduled tribes.

Tenth Schedule: Provisions relating to disqualification of the members of Parliament and State

Legislatures on the ground of defection. This schedule was added by the 52nd Amendment Act of 1985,

also known as Anti-defection Law.

Q 60.A

Statement 1 is correct: The Commission consists of a State Chief Information Commissioner and not

more than ten State Information Commissioners. They are appointed by the Governor on the

recommendation of a committee consisting of the Chief Minister as Chairperson, the Leader of

Opposition in the Legislative Assembly and a State Cabinet Minister nominated by the Chief Minister.

Statement 2 is not correct: The Governor can remove the State Chief Information Commissioner or any

State Information Commissioner on the ground of proved misbehaviour or incapacity. In these cases, the

Governor has to refer the matter to the Supreme Court for an enquiry. If the Supreme Court, after the

enquiry, upholds the cause of removal and advises so, then the Governor can remove him.

Page 36: xamlibrary.files.wordpress.com · 2018-09-01 · 5 ©Vision IAS 17. The Indian Councils Act of 1909 provided for which of the following provisions? 1. Non-official majority in Central

17 www.visionias.in ©Vision IAS

Q 61.C

The National Human Rights Commission is a multi-member body consisting of a chairman and four

members. The chairman should be a retired chief justice of India, and members should be serving or

retired judges of the Supreme Court, a serving or retired chief justice of a high court and two persons

having knowledge or practical experience with respect to human rights. In addition to these full-time

members, the commission also has four ex-officio members—the chairmen of the National Commission

for Minorities, the National Commission for SCs, the National Commission for STs and the National

Commission for Women.

Q 62.B

The National Institution for Transforming India, also called NITI Aayog, was formed via a resolution of

the Union Cabinet on January 1, 2015. NITI Aayog is the premier policy ‗Think Tank‘ of the Government

of India, providing both directional and policy inputs. While designing strategic and long term policies

and programmes for the Government of India, NITI Aayog also provides relevant technical advice to the

Centre and States.

Q 63.D

The procedure for the amendment of the Constitution as laid down in Article 368 is as follows:

o An amendment of the Constitution can be initiated only by the introduction of a bill for the purpose in

either House of Parliament and not in the state legislatures.

o The bill can be introduced either by a minister or by a private member and does not require prior

permission of the president. Hence, statement 1 is not correct.

o The bill must be passed in each House by a special majority, that is, a majority (that is, more than 50

per cent) of the total membership of the House and a majority of two-thirds of the members of the

House present and voting. Hence, statement 2 is not correct.

o Each House must pass the bill separately. In case of a disagreement between the two Houses, there is

no provision for holding a joint sitting of the two Houses for the purpose of deliberation and passage

of the bill.

o If the bill seeks to amend the federal provisions of the Constitution, it must also be ratified by the

legislatures of atleast half of the states by a simple majority, that is, a majority of the members of the

House present and voting.

o After duly passed by both the Houses of Parliament and ratified by the state legislatures, where

necessary, the bill is presented to the president for assent.

o The President must give his assent to the bill. He can neither withhold his assent to the bill nor return

the bill for reconsideration of the Parliament.

Q 64.A

The Fundamental Duties in the Indian Constitution were inspired by the Constitution of erstwhile USSR.

Notably, none of the Constitutions of major democratic countries like USA, Canada, France, Germany,

Australia and so on specifically contain a list of duties of citizens. The socialist countries, on the contrary,

gave equal importance to the fundamental rights and duties of their citizens. Hence, the Constitution of

erstwhile USSR declared that the citizen‘s exercise of their rights and freedoms was inseparable from the

performance of their duties and obligations.

Q 65.A

Article 13 declares that all laws that are inconsistent with or in derogation of any of the

fundamental rights shall be void. In other words, it explicitly provides for the doctrine of judicial

review. This power has been conferred on the Supreme Court (Article 32) and the high courts (Article

226) that can declare a law unconstitutional and invalid on the ground of contravention of any of the

Fundamental Rights.

Article 50 provides for separation of Judiciary from the Executive.

Q 66.D

Article 35 lays down that the power to make laws, to give effect to certain specified fundamental

rights shall vest only in the Parliament and not in the state legislatures. This provision ensures that

Page 37: xamlibrary.files.wordpress.com · 2018-09-01 · 5 ©Vision IAS 17. The Indian Councils Act of 1909 provided for which of the following provisions? 1. Non-official majority in Central

18 www.visionias.in ©Vision IAS

there is uniformity throughout India with regard to the nature of those fundamental rights and punishment

for their infringement. In this direction, Article 35 contains the following provisions:

o The Parliament shall have (and the legislature of a state shall not have) power to make laws with

respect to the following matters:

Prescribing residence as a condition for certain employments or appointments in a state or

union territory or local authority or other authority (Article 16).

Empowering courts other than the Supreme Court and the high courts to issue directions, orders

and writs of all kinds for the enforcement of fundamental rights (Article 32).

Restricting or abrogating the application of Fundamental Rights to members of armed

forces, police forces, etc. (Article 33).

Indemnifying any government servant or any other person for any act done during the operation

of martial law in any area (Article 34).

o Parliament shall have (and the legislature of a state shall not have) powers to make laws for

prescribing punishment for those acts that are declared to be offences under the fundamental rights.

These include the following:

Untouchability (Article 17).

Traffic in human beings and forced labour (Article 23).It should be noted that Article 35

extends the competence of the Parliament to make a law on the matters specified above, even

though some of those matters may fall within the sphere of the state legislatures (i.e., State List).

Hence, none of the given options is correct.

Q 67.A

Statement 1 is correct: State Election Commissions is entrusted to conduct elections to Panchayats

through 73rd and urban local bodies through 74th Constitutional amendment acts.

Statement 2 is correct: The functions of delimitation, reservation and rotation of Panchayats seats is

vested in State Election Commission. As per the recommendations of National Commission to Review the

Working of the Constitution (NCRWC), the functions of delimitation, reservation and rotation of seats

should be vested in a Delimitation Commission and not in the State Election Commission.

Statement 3 is not correct: At present, there is no clear-cut provision regarding the submission of reports

by State Election Commission. NCRWC also recommended that the State Election Commission should

submit its annual or special reports to the Election Commission of India and to the Governor.

Q 68.A

Eighth Schedule contains Languages recognized by the Constitution. Originally, it had 14 languages but

presently there are 22 languages.

They are: Assamese, Bengali, Bodo, Dogri (Dongri), Gujarati, Hindi, Kannada, Kashmiri, Konkani,

Mathili (Maithili), Malayalam, Manipuri, Marathi, Nepali, Odia, Punjabi, Sanskrit, Santhali, Sindhi,

Tamil, Telugu and Urdu.

Sindhi was added by the 21st Amendment Act of 1967.

Konkani, Manipuri and Nepali were added by the 71st Amendment Act of 1992.

Bodo, Dongri, Maithili and Santhali were added by the 92nd Amendment Act of 2003.

Oriya was renamed as ‗Odia‘ by the 96th Amendment Act of 2011.

Q 69.D

Fundamental Rights are different from other rights available to us. While ordinary legal rights are

protected and enforced by ordinary law, Fundamental Rights are protected and guaranteed by the

constitution of the country. But Fundamental Rights are not absolute, they are imposed with

reasonable restrictions. Ordinary rights may be changed by the legislature by ordinary process of law

making, but a fundamental right may only be changed by amending the Constitution itself. So the

Fundamental Rights are difficult to amend.

Hence, both the statements are not correct.

Q 70.D

The Constitution states that the word ‗law‘ used in Article 13 includes Ordinance, order, legislations, rules

and even customs that have force of law within Indian territory

Page 38: xamlibrary.files.wordpress.com · 2018-09-01 · 5 ©Vision IAS 17. The Indian Councils Act of 1909 provided for which of the following provisions? 1. Non-official majority in Central

19 www.visionias.in ©Vision IAS

The Supreme Court in Shankari Prasad v/s Union of India (1951) ruled that the power of the Parliament to

amend the Constitution under Article 368 also includes power to amend Fundamental Rights. Hence, the

word ‗law‘ used in Article 13 includes ordinary laws and not Constitutional Amendment Acts.

Q 71.C

Statement 1 is not correct: Though the word ―Secular‖ was not added to the Preamble initially, the

Indian Constitution has been secular from the beginning. It has Freedom of Religion (Art 25-28) and

Protection of rights of minorities (Art 29-30) as Fundamental Rights. However, Indian secularism is

different from the Western concept of secularism, where religion is treated as a personal matter and there

is a strict separation between religion and State. In India, State can regulate economic, political and

secular activities related to religious practices, for example- throwing open Hindu religious institutions to

all sections.

Statement 2 is correct: As per Art 29(2), no citizen can be denied entry into any educational institute

maintained by the State only on the grounds of religion, race, caste or any of them.

Statement 3 is not correct: The Freedom of Religion is not absolute. The Constitution provides for

certain limitations on them. For example, every religious denomination has the right to establish and

maintain institutions for religious and charitable purposes. But this is subject to restrictions of public

order, morality and health.

Q 72.C

Mandamus is a command issued by the court to a public official asking him to perform his official

duties that he has failed or refused to perform. It can also be issued against any public body, a

corporation, an inferior court, a tribunal or government for the same purpose. The writ of mandamus

cannot be issued (a) against a private individual or body; (b) to enforce departmental instruction that

does not possess statutory force; (c) when the duty is discretionary and not mandatory; (d) to enforce a

contractual obligation; (e) against the President of India or the state Governors; and (f) against the Chief

Justice of a High Court acting in judicial capacity.

Q 73.B

Statement 1 is not correct: A proclamation of President‘s Rule may be revoked by the President at any

time by a subsequent proclamation. Such a proclamation does not require the parliamentary approval.

Statement 2 is correct: The 44th Amendment Act of 1978 introduced a new provision to put restraint on

the power of Parliament to extend a proclamation of President‘s Rule beyond one year. Thus, it provided

that, beyond one year, the President‘s Rule can be extended by six months at a time only when the

following two conditions are fulfilled: 1. a proclamation of National Emergency should be in operation in

the whole of India, or in the whole or any part of the state; and 2. the Election Commission must certify

that the general elections to the legislative assembly of the concerned state cannot be held on account of

difficulties.

Q 74.D

Statement 1 is not correct: The Supreme Court (under Article 32) and the high courts (under Article

226) can issue the writs of habeas corpus, mandamus, prohibition, certiorari and quo-warranto. Further,

the Parliament (under Article 32) can empower any other court to issue these writs. Since no such

provision has been made so far, only the Supreme Court and the high courts can issue the writs and not

any other court.

Statement 2 is not correct: The Supreme Court can issue writs only for the enforcement of fundamental

rights whereas a high court can issue writs not only for the enforcement of Fundamental Rights but also

for any other purpose. The expression ‗for any other purpose‘ refers to the enforcement of an ordinary

legal right. Thus, the writ jurisdiction of the Supreme Court, in this respect, is narrower than that of high

court.

Q 75.B

Statement 1 is not correct: The First Amendment Act, 1951 added three more grounds of restriction on

Freedom of Speech and Expression - public order, friendly relations with foreign states, incitement to an

offence. Also it made restrictions 'reasonable' and thus justiciable in nature.The Parliament has been

Page 39: xamlibrary.files.wordpress.com · 2018-09-01 · 5 ©Vision IAS 17. The Indian Councils Act of 1909 provided for which of the following provisions? 1. Non-official majority in Central

20 www.visionias.in ©Vision IAS

authorised by law to make laws regarding 'reasonable restrictions' under Freedom of Speech and

Expression.

Statement 2 is correct: Defamation one of the reasonable restrictions under Freedom of speech and

expression is of two types- criminal defamation (public and private interest) and civil defamation (private

interest).

Q 76.D

National Commission for Scheduled Tribes (STs) is a constitutional body in the sense that it is

directly established by Article 338-A of the Constitution.

The Central Bureau of Investigation (CBI) was set up in 1963 by a resolution of the Ministry of Home

Affairs. It derives its powers from the Delhi Special Police Establishment Act, 1946. The CBI is the main

investigating agency of the Central Government. It plays an important role in the prevention of corruption

and maintaining integrity in administration.

The National Institution for Transforming India, also called NITI Aayog, was formed via a resolution

of the Union Cabinet on January 1, 2015. NITI Aayog is the premier policy ‗Think Tank‘ of the

Government of India, providing both directional and policy inputs. While designing strategic and long

term policies and programmes for the Government of India, NITI Aayog also provides relevant technical

advice to the Centre and States.

Q 77.A

Freedom of speech and expression is guaranteed under Article 19(1) of Indian constitution. It

implies that every citizen has the right to express his views, opinions, belief and convictions. The

Supreme Court held that the freedom of speech and expression includes the following -:

o Right to propagate one's views as well as views of others.

o Freedom of press

o Freedom of Commercial adverstisements

o Right against tapping of telephonic conversation

o Right to telecast

o Freedom of silence

o Right to know about government activities

o Right against imposition of pre-censorship

o Right to demonstration or picketing but not right to strike

Q 78.D

Statement 1 is not correct: Though the Indian Constitution is federal and envisages a dual polity (Centre

and states), it provides for only a single citizenship, that is, the Indian citizenship (like in Canada). The

citizens in India owe allegiance only to the Union. There is no separate state citizenship. The other federal

states like USA and Switzerland, on the other hand, adopted the system of double citizenship.

Statement 2 is not correct: In India both a citizen by birth as well as a naturalised citizen are eligible for

the office of President while in USA, only a citizen by birth and not a naturalised citizen is eligible for the

office of President.

Q 79.C

The Constituent Assembly was constituted in November 1946 under the scheme formulated by the

Cabinet Mission Plan.

The Assembly was made a fully sovereign body, which could frame any Constitution it pleased. The act

empowered the Assembly to abrogate or alter any law made by the British Parliament in relation to India.

It worked as a legislative body i.e. two separate functions were assigned to the Assembly - making of a

constitution for free India and enacting of ordinary laws for the country. Thus, the Assembly became the

first Parliament of free India (Dominion Legislature). Whenever the Assembly met as the Constituent

body it was chaired by Dr. Rajendra Prasad and when it met as the legislative body, it was chaired by G V

Mavlankar. These two functions continued till November 26, 1949, when the task of making the

Constitution was over.

Although the Constituent Assembly was not directly elected by the people of India on the basis of

adult franchise, the Assembly comprised representatives of all sections of Indian Society—Hindus,

Page 40: xamlibrary.files.wordpress.com · 2018-09-01 · 5 ©Vision IAS 17. The Indian Councils Act of 1909 provided for which of the following provisions? 1. Non-official majority in Central

21 www.visionias.in ©Vision IAS

Muslims, Sikhs, Parsis, Anglo–Indians, Indian Christians, SCs, STs including women of all these

sections.

Q 80.D

In 1973, the Supreme Court ruled in Kesavananda Bharati case that there is a basic structure of the

Constitution and nobody—not even the Parliament (through amendment)—can violate the basic

structure. The Court did two more things. First, it said that right to property (the disputed issue) was not

part of basic structure and therefore could be suitably abridged. Secondly, the Court reserved to itself

the right to decide whether various matters are part of the basic structure of the Constitution. This

case is perhaps the best example of how judiciary uses its power to interpret the Constitution.

Q 81.C

Statement 1 is not correct: The Constitution makes a provision for the establishment of a Joint State

Public Service Commission (JSPSC) for two or more states. While the UPSC and the SPSC are created

directly by the Constitution, a JSPSC can be created by an act of Parliament on the request of the state

legislatures concerned.

Statement 2 is correct: The chairman and members of a JSPSC are appointed by the president. They hold

office for a term of six years or until they attain the age of 62 years, whichever is earlier.

Statement 3 is not correct: A JPSC presents its annual performance report to each of the concerned state

governors. Each governor places the report before the state legislature.

Q 82.D

Statement 1 is correct. Attorney General is the highest law officer in the country. The Attorney General

(AG) is appointed by the President. He must be a person who is qualified to be appointed a judge of the

Supreme Court.

Statement 2 is not correct. The remuneration of the AG is not fixed by the Constitution. He receives

such remuneration as the President may determine.

Statement 3 is correct. He has the right to speak and to take part in the proceedings of both the Houses of

Parliament or their joint sitting and any committee of the Parliament of which he may be named a member

but without a right to vote.

Q 83.D

In addition to the making of the Constitution and enacting of ordinary laws, the Constituent Assembly

also performed the following functions:

o It ratified the India’s membership of the Commonwealth in May 1949.

o It adopted the national flag on July 22, 1947.

o It adopted the national anthem on January 24, 1950.

o It adopted the national song on January 24, 1950.

o It elected Dr Rajendra Prasad as the first President of India on January 24, 1950.

Q 84.C

In the Constituent Assembly speech by Dr B.R. Ambedkar on Friday, the 25th November, 1949, he said,

―What does social democracy mean? It means a way of life which recognizes liberty, equality and

fraternity as the principles of life. These principles of liberty, equality and fraternity as the principles of

life. These principles of liberty, equality and fraternity are not to be treated as separate items in a trinity.

They form a union of trinity in the sense that to divorce one from the other is to defeat the very purpose of

democracy. Liberty cannot be divorced from equality, equality cannot be divorced from liberty. Nor can

liberty and equality be divorced from fraternity. Without equality, liberty would produce the

supremacy of the few over the many. Equality without liberty would kill individual initiative.

Without fraternity, liberty would produce the supremacy of the few over the many. Equality without

liberty would kill individual initiative. Without fraternity, liberty and equality could not become a natural

course of things. It would require a constable to enforce them. Without fraternity equality and liberty will

be no deeper than coats of paint.‖

Page 41: xamlibrary.files.wordpress.com · 2018-09-01 · 5 ©Vision IAS 17. The Indian Councils Act of 1909 provided for which of the following provisions? 1. Non-official majority in Central

22 www.visionias.in ©Vision IAS

Q 85.B

Major Committees of the Constituent Assembly under Jawaharlal Nehru were:

o Union Powers Committee

o Union Constitution Committee

o States Committee (Committee for negotiating with States)

Major Committees of the Constituent Assembly under Sardar Patel:

o Provincial Constitution Committee

o Advisory Committee on Fundamental Rights, Minorities and Tribal and Excluded Areas

Major Committees of the Constituent Assembly under Dr. Rajendra Prasad:

o Rules of Procedure Committee

o Steering Committee

Q 86.C

National Commission for Minorities fall under Minorities Ministry of Minority Affairs.

National Commission for Women falls under Ministry of Women and Child Development.

National Commission for STs falls under Ministry of Tribal Affairs.

Hence, only pairs 2 and 3 are correct.

Q 87.A

Pitt's India Act of 1784 distinguished between the commercial and political functions of the Company. It

allowed the Court of Directors to manage the commercial affairs but created a new body called Board of

Control to manage the political affairs. Thus, it established a system of double government, that is,

separate body for commercial and administrative functions of the East India Company. Hence, statement

2 is not correct.

Features of the Charter Act of 1833:

It made the Governor-General of Bengal as the Governor-General of India and vested in him all civil and

military powers. Lord William Bentick was the first governor-general of India.

It deprived the governor of Bombay and Madras of their legislative powers. The Governor-General of

India was given exclusive legislative powers for the entire British India. Hence, statement 1 is correct.

It ended the activities of the East India Company as a commercial body, which became a purely

administrative body. It provided that the company‘s territories in India were held by it ‗in trust for His

Majesty, His heirs and successors‘.

The Charter Act of 1833 attempted to introduce a system of open competition for selection of civil

servants. However, this provision was negated after opposition from the Court of Directors. Finally, the

Charter Act of 1853 introduced an open competition system of selection and recruitment of civil servants.

Hence, statement 3 is not correct.

Q 88.C

Article 15 provides that the State shall not discriminate against any citizen on grounds only of religion,

race, caste, sex or place of birth. The two crucial words in this provision are ‗discrimination‘ and ‗only‘.

The word ‗discrimination‘ means ‗to make an adverse distinction with regard to‘ or ‗to distinguish

unfavourably from others‘. The use of the word ‗only‘ connotes that discrimination on other grounds is

not prohibited.

Q 89.D

Both the statements are not correct.

During an emergency, the Central Government becomes all-powerful and the states go into the total

control of the centre. It converts the federal structure into a unitary one without a formal amendment of

the Constitution. This kind of transformation of the political system from federal (during normal

times) to unitary (during emergency) is not borrowed from any Constitution of the world.

The feature of suspension of Fundamental Rights during Emergency has been borrowed from the

Weimar Constitution of Germany and hence, not a unique feature of the Indian Constitution.

Page 42: xamlibrary.files.wordpress.com · 2018-09-01 · 5 ©Vision IAS 17. The Indian Councils Act of 1909 provided for which of the following provisions? 1. Non-official majority in Central

23 www.visionias.in ©Vision IAS

Q 90.C

If any foreign territory becomes a part of India, the Government of India specifies the persons who

among the people of the territory shall be the citizens of India. Such persons become the citizens of

India from the notified date. For example, when Pondicherry became a part of India, the Government of

India issued the Citizenship (Pondicherry) Order, 1962, under the Citizenship Act, 1955.

Q 91.D

Statement 1 is not correct: ‗Territory of India‘ is a wider expression than the ‗Union of India‘ because

the latter includes only states while the former includes not only the states but also union territories and

territories that may be acquired by the Government of India at any future time.

Statement 2 is not correct: Being a sovereign state, India can acquire foreign territories according to the

modes recognised by international law, i.e., cession (following treaty, purchase, gift, lease or plebiscite),

occupation (hitherto unoccupied by a recognised ruler), conquest or subjugation.

Q 92.D

The Citizenship Act, 1955, prescribes three ways of losing citizenship whether acquired under the Act or

prior to it under the Constitution. These are:

o By Renunciation: Any citizen of India of full age and capacity can make a declaration renouncing his

Indian citizenship. Upon the registration of that declaration, that person ceases to be a citizen of India.

o By Termination: When an Indian citizen voluntarily acquires the citizenship of another country, his

Indian citizenship automatically terminates. Hence, statement 1 is correct.

o By Deprivation: It is a compulsory termination of Indian citizenship by the Central government, if:

the citizen has obtained the citizenship by fraud. Hence, statement 3 is correct.

the citizen has shown disloyalty to the Constitution of India.

the citizen has unlawfully traded or communicated with the enemy during a war. Hence,

statement 2 is correct.

the citizen has, within five years after registration or naturalisation, been imprisoned in any

country for two years; and

the citizen has been ordinarily resident out of India for seven years continuously.

Q 93.C

Dr. Ambedkar considered the right to constitutional remedies as ‘heart and soul of the

constitution’. It is so because this right gives a citizen the right to approach a High Court or the Supreme

Court to get any of the fundamental rights restored in case of their violation. The Supreme Court and the

High Courts can issue orders and give directives to the government for the enforcement of rights.

Q 94.D

The Chief Information Commissioner and an Information Commissioner hold office for a term of 5

years or until they attain the age of 65 years, whichever is earlier. They are not eligible for

reappointment.

They should be persons of eminence in public life with wide knowledge and experience in law, science

and technology, social service, management, journalism, mass media or administration and governance.

They should not be a Member of Parliament or Member of the Legislature of any State or Union

Territory. They should not hold any other office of profit or connected with any political party or

carrying on any business or pursuing any profession.

Hence, all the statements are correct.

Q 95.D

The Constitution of India was adopted on 26th November 1949 and formally started from January 1950.

Since then, the constitution continues to function as the framework in which the government of our

country operates.

In the actual working of the Constitution, there has been enough flexibility of interpretations. Both

political practice and judicial rulings have shown maturity and flexibility in implementing the

Constitution. Changes have regularly been made in order to reflect the contemporary aspects of

political philosophy and aspirations of the society. The Judiciary has contributed by interpreting

various provisions concerning right to education, right to life and liberty and the right to form and manage

Page 43: xamlibrary.files.wordpress.com · 2018-09-01 · 5 ©Vision IAS 17. The Indian Councils Act of 1909 provided for which of the following provisions? 1. Non-official majority in Central

24 www.visionias.in ©Vision IAS

minority educational institutions. Political parties, political leaders, the government, and the Parliament,

accepted the idea of inviolable basic structure introduced in the Kesavananda case. Even when there was

talk about ‗review‘ of the Constitution, it was understood that the exercise could not cross the limits set by

the theory of the basic structure.

The basic framework of the Constitution continues to suit our country. The provisions of the

constitution reflect efforts to tackle the problems that the society was facing at the time of making of the

constitution. At the same time, the constitution was envisioned to be a document that provides the

framework of the government for the future as well. Therefore, the constitution allows for a response to

the challenges that may arise in the future.

The makers of the Constitution placed the Constitution above ordinary law and expected the future

generations will respect this document. At the same time, they recognised that in the future, it may

require modifications. They understood that Constitution must be amended if required but it must be

protected from unnecessary and frequent changes as well.

The constitution is a sacred document but it may require changes from time to time as well.

Therefore, the Constitution is not a static document or not the final word about everything and is not

unalterable.

Q 96.A

Pressure groups are organisations that attempt to influence government policies. But unlike political

parties, pressure groups do not aim to directly control or share political power. These organisations are

formed when people with common occupation, interest, aspirations or opinions come together in order to

achieve a common objective.

We often hear the word people’s movement to describe many forms of collective action: Narmada

Bachao Andolan, Movement for Right to Information, Anti-liquor Movement, Women‘s Movement,

Environmental Movement.

Like an interest group, a people’s movement also attempts to influence politics rather than directly

taking part in electoral competition. But unlike the interest groups, people’s movements have a

loose organisation. Their decision making is more informal and flexible. They depend much more

on spontaneous mass participation than an interest group.

Q 97.D

The Constitution of India provides certain rights in form of Equalities or Equal treatment to its citizens.

These rights are incorporated as Fundamental Rights in the Constitution. These Equalities include:

o Equality of Status: Equality of Status implies that all citizens have the same status in certain

respects, including civil rights, freedom of speech, property rights etc. Articles 17 and 18 aim at

providing Equality of Status. Article 17 abolishes Untouchability and Article 18 prohibits awarding of

titles.

o Equality of Opportunity: Equality of Opportunity arises from the similar treatment of all people,

unhindered by prejudices or preferences, except when particular distinctions can be explicitly

justified. It means that chances for advancement should be open to everybody interested. The

Constitution aims to achieve this by providing reservation in educational institutes and public

employment (Articles 15 and 16 respectively) to the disadvantaged sections of society.

o Equality before Law: This is a fundamental principle included in Rule of Law. It implies that every

citizen is equal in the eyes of law. Article 14 explicitly provides for Equality before law and Equal

protection of laws.

o Equality of Outcomes: It describes a state in which people have approximately the same material

wealth and income, or in which the general economic conditions of their lives are alike. The

Constitution does not provide for Equality of Outcomes.

Hence, options 1, 2 and 4 are correct but option 3 is not correct.

Q 98.D

The Indian Penal Code (IPC) declares the imputations and assertions prejudicial to national integration

as punishable offences. (Article 51(a) (c) to uphold and protect the sovereignty, unity and integrity of

India;)

Page 44: xamlibrary.files.wordpress.com · 2018-09-01 · 5 ©Vision IAS 17. The Indian Councils Act of 1909 provided for which of the following provisions? 1. Non-official majority in Central

25 www.visionias.in ©Vision IAS

The Representation of People Act (1951) provides for the disqualification of members of the Parliament

or a state legislature for indulging in corrupt practice, that is, soliciting votes on the ground of religion or

promoting enmity between different sections of people on grounds of caste, race, language, religion and

so on. (Article 51(a) (e) to promote harmony and the spirit of common brotherhood amongst all the

people of India transcending religious, linguistic and regional or sectional diversities and to renounce

practices derogatory to the dignity of women;)

The Forest (Conservation) Act of 1980 checks indiscriminate deforestation and diversion of forest land

for non-forest purposes. These are in line with the (Article 51(a) (g) to protect and improve the natural

environment including forests, lakes, rivers and wildlife and to have compassion for living creatures;)

Q 99.C

Statement 1 is not correct: It is concerned with the recruitment to the all-India services and Central

services—group A and group B and advises the government, when consulted, on promotion and

disciplinary matters. It is not concerned with the classification of services, pay and service conditions,

cadre management, training, etc which is managed by the Department of Personnel and Training of

government of India..

Statement 2 is not correct: The UPSC presents, annually, to the President a report on its performance.

The President places this report before both the Houses of Parliament, along with a memorandum

explaining the cases where the advice of the Commission was not accepted and the reasons for such non-

acceptance.

Statement 3 is correct: The additional functions relating to the services of the Union can be conferred on

UPSC by the Parliament. It can also place the personnel system of any authority, corporate body or public

institution within the jurisdiction of the UPSC. Hence the jurisdiction of UPSC can be extended by an Act

made by the Parliament.

Q 100.A

Even though the Indian Parliamentary System is largely based on the British pattern, there are some

fundamental differences between the two:

o The British system is based on the doctrine of the sovereignty of Parliament, while the Parliament is

not supreme in India and enjoys limited and restricted powers due to a written Constitution, federal

system, judicial review and fundamental rights. Hence, statement 1 is correct

o The Indian State has an indirectly elected head (republic) while the British State has a hereditary

head (monarchy). Hence, statement 2 is not correct.

o In Britain, the prime minister should be a member of the Lower House (House of Commons) of the

Parliament. In India, the prime minister may be a member of any of the two Houses of Parliament.

o Usually, the members of Parliament alone are appointed as ministers in Britain. In India, a person who

is not a member of Parliament can also be appointed as minister, but for a maximum period of six

months.

o Britain has the system of legal responsibility of the minister while India has no such system. Unlike in

Britain, the ministers in India are not required to countersign the official acts of the Head of the State.

Copyright © by Vision IAS All rights are reserved. No part of this document may be reproduced, stored in a retrieval system or transmitted in any form or by any means, electronic, mechanical, photocopying, recording or otherwise, without prior permission of Vision IAS.